*NURSING > EXAM > NUR 4290 HESI EXAM QUESTIONS AND ANSWERS (LATEST 100% ) (All)

NUR 4290 HESI EXAM QUESTIONS AND ANSWERS (LATEST 100% )

Document Content and Description Below

NUR 4290 HESI EXAMNUR 4290 HESI EXAM Following a lumbar puncture, a client voices several complaints. What complaint indicated to the nurse that the client is experiencing a complication? A. “I h... ave a headache that gets worse when I sit up” 
 B. “I am having pain in my lower back when I move my legs” 
 C. “My throat hurts when I swallow” 
 D. “I feel sick to my stomach and am going to throw up” 
 A mother brings her 6-year-old child, who has just stepped on a rusty nail, to the pediatrician’s office. Upon inspection, the nurse notes that the nail went through the shoe and pierced the bottom of the child’s foot. Which action should the nurse implement first? E. Cleanse the foot with soap and water and apply an antibiotic ointment 
 F. Provide teaching about the need for a tetanus booster within the next 72 hours. 
 G. have the mother check the child's temperature q4h for the next 24 hours 
 H. transfer the child to the emergency department to receive a gamma globulin 
injection 
 A 26-year-old female client is admitted to the hospital for treatment of a simple goiter, and levothyroxine sodium (Synthroid) is prescribed. Which symptoms indicate to the nurse that the prescribed dosage is too high for this client? The client experiences A. Bradycardia and constipation 
 B. Lethargy and lack of appetite 
 C. Muscle cramping and dry, flushed skin 
 D. Palpitations and shortness of breath The pathophysiological mechanism are responsible for ascites related to liver failure? (Select all that apply 
 A. Fluid shifts from intravascular to interstitial area due to decreased serum protein B. Increased hydrostatic pressure in portal circulation increases fluid shifts into abdomen C. Increased circulating aldosterone levels that increase sodium and water retention The nurse notes that a client has been receiving hydromorphone (Dilaudid) every six 
hours for four days. What assessment is most important for the nurse to complete? A. Auscultate the client's bowel sounds 
 B. Observe for edema around the ankles 
 C. Measure the client’s capillary glucose level 
 D. Count the apical and radial pulses simultaneously 
 After a third hospitalization 6 months ago, a client is admitted to the hospital with ascites and malnutrition. The client is drowsy but responding to verbal stimuli and reports recently spitting up blood. What assessment finding warrants immediate intervention by the nurse? A. Capillary refill of 8 seconds 
 B. bruises on arms and legs 
 C. round and tight abdomen 
 D. pitting edema in lower legs 
 The client with which type of wound is most likely to need immediate intervention by the 
nurse? A. Laceration 
 B. Abrasion 
 C. Contusion 
 D. Ulceration 
 When caring for a client who has acute respiratory distress syndrome (ARDS), the nurse elevates the head of the bed 30 degrees. What is the reason for this intervention? A. To reduce abdominal pressure on the diaphragm 
 B. to promote retraction of the intercostal accessory muscle of respiration 
 C. to promote bronchodilation and effective airway clearance 
 D. to decrease pressure on the medullary center which stimulates breathing 
 When assessing a mildly obese 35-year-old female client, the nurse is unable to locate the gallbladder when palpating below the liver margin at the lateral border of the rectus abdominal muscle. What is the most likely explanation for failure to locate the gallbladder by palpation? A. The client is too obese 
 B. Palpating in the wrong abdominal quadrant 
 C. Deeper palpation technique is needed 
 D. The gallbladder is normal 
 A woman with an anxiety disorder calls her obstetrician’s office and tells the nurse of increased anxiety since the normal vaginal delivery of her son three weeks ago. Since she is breastfeeding, she stopped taking her antianxiety medications, but thinks she may need to start taking them again because of her increased anxiety. What response is best for the nurse to provide this woman? A. describe the transmission of drugs to the infant through breast milk 
 B. encourage her to use stress relieving alternatives, such as deep breathing exercises C. Inform her that some antianxiety medications are safe to take while breastfeeding D. Explain that anxiety is a normal response for the mother of a 3-week-old 
 An older male client with a history of type 1 diabetes has not felt well the past few days and arrives at the clinic with abdominal cramping and vomiting. He is lethargic, moderately, confused, and cannot remember when he took his last dose of insulin or ate last. What action should the nurse implement first? A. Start an intravenous (IV) infusion of normal saline 
 B. obtain a serum potassium level 
 C. administer the client's usual dose of insulin 
 D. assess pupillary response to light 
 A client who received multiple antihypertensive medications experiences syncope due to a drop-in blood pressure to 70/40. What is the rationale for the nurse’s decision to hold the client’s scheduled antihypertensive medication? A. increased urinary clearance of the multiple medications has produced diuresis and lowered the blood pressure 
 B. the antagonistic interaction among the various blood pressure medications has reduced their effectiveness 
 C. the synergistic effect of the multiple medications has resulted in drug toxicity and resulting hypotension 
 D. The additive effect of multiple medications has caused the blood pressure too low Which client is at the greatest risk for developing delirium? A. An adult client who cannot sleep due to constant pain. 
 B. an older client who attempted 1 month ago 
 C. a young adult who takes antipsychotic medications twice a day 
 D. a middle-aged woman who uses a tank for supplemental oxygen 
 Which intervention should the nurse include in a long-term plan of care for a client with Chronic Obstructive Pulmonary Disease (COPD)? A. Reduce risks factors for infection 
 B. Administer high flow oxygen during sleep 
 C. Limit fluid intake to reduce secretions 
 D. Use diaphragmatic breathing to achieve better exhalation 
 Which location should the nurse choose as the best for beginning a screening program for hypothyroidism? A. A business and professional women's group. B. An African-American senior citizens center 
 C. A daycare center in a Hispanic neighborhood 
 D. An after-school center for Native-American teens 
 A female client has been taking a high dose of prednisone, a corticosteroid, for several months. After stopping the medication abruptly, the client reports feeling “very tired”. Which nursing intervention is most important for the nurse to implement? A. Measure vital signs B. Auscultate breath sounds 
 C. Palpate the abdomen 
 D. Observe the skin for bruising 
 A male client reports the onset of numbness and tingling in his fingers and around his mouth. Which lab is important for the nurse to review before contacting the health care provider? A. capillary glucose 
 B. urine specific gravity 
 C. Serum calcium 
 D. white blood cell count 
 What explanation is best for the nurse to provide a client who asks the purpose of using the log-rolling technique for turning? A. working together can decrease the risk for back injury 
 B. The technique is intended to maintain straight spinal alignment. 
 C. Using two or three people increases client safety. 
 D. turning instead of pulling reduces the likelihood of skin damage 
 A client is admitted to isolation with the diagnosis of active tuberculosis. Which infection control measures should the nurse implement? A. Negative pressure environment 
 B. contact precautions 
 C. droplet precautions 
 D. protective environment 
 A client is admitted to the intensive care unit with diabetes insipidus due to a pituitary gland tumor. Which potential complication should the nurse monitor closely? A. Hypokalemia 
 B. Ketonuria. 
 C. Peripheral edema 
 D. Elevated blood pressure The nurse is assessing a 3-month-old infant who had a pylorotomy yesterday. This child should be medicated for pain based on which findings? Select all that apply: A. Restlessness B. Clenched Fist C. Increased pulse rate D. Increased respiratory rate. E. Increased temperature 
 F. Peripheral pallor of the skin 
 The nurse who is working on a surgical unit receives change of shift report on a group of 
clients for the upcoming shift. A client with which condition requires the most immediate attention by the nurse? 
 A. Gunshot wound three hours ago with dark drainage of 2 cm noted on the dressing. 
 B. Mastectomy 2 days ago with 50 ml bloody drainage noted in the Jackson-pratt 
drain. 
 C. Collapsed lung after a fall 8h ago with 100 ml blood in the chest tube collection 
container 
 D. Abdominal-perineal resection 2 days ago with no drainage on dressing who has fever and chills In caring for a client with Cushing syndrome, which serum laboratory value is most 
important for the nurse to monitor? A. Lactate B. Glucose
 C. Hemoglobin D. Creatinine Which type of Leukocyte is involved with allergic responses and the destruction of parasitic worms? 
 A. Neutrophils B. Lymphocytes C. Eosinophils (Eosinophils are involved in allergic response and destruction of parasitic worms) D. Monocytes A client who sustained a head injury following an automobile collision is admitted to the hospital. The nurse include the client’s risk for developing increased intracranial pressure (ICP) in the plan of care. Which signs indicate to the nurse that ICP has increased? A. Increased Glasgow coma scale score. 
 B. Nuchal rigidity and papilledema. 
 C. Confusion and papilledema 
 D. Periorbital ecchymosis. 
 Papilledema is always an indicator of increased ICP. Confusion is usually the first sign The nurse is developing an educational program for older clients who are being discharged with new antihypertensive medications. The nurse should ensure that the educational materials include which characteristics? Select all that apply A. Written at a twelfth-grade reading level B. Contains a list with definitions of unfamiliar terms C. Uses common words with few Syllables D. Printed using a 12 point type font E. Uses pictures to help illustrate complex ideas A client with a chronic health problem has difficulty ambulating short distance due to generalized weakness, but is able to bear weight on both legs. To assist with ambulation and provide the greatest stability, what assistive device is best for this client? A. Crutches with 2 point gait. B. Crutches with 3 point gait. C. Crutches with 4 point gait. D. A quad cane A client with leukemia undergoes a bone marrow biopsy. The client’s laboratory values indicate the client has thrombocytopenia. Based on this data, which nursing assessment is most important following the procedure? A. Observe aspiration site. B. Assess body temperature C. Monitor skin elasticity D. Measure urinary output When implementing a disaster intervention plan, which intervention should the nurse implement first? A. Initiate the discharge of stable clients from hospital units B. Identify a command center where activities are coordinated C. Assess community safety needs impacted by the disaster D. Instruct all essential off-duty personnel to report to the facility The nurse is auscultating a client’s lung sounds. Which description should the nurse use to document this sound? NO SOUND A. High pitched or fine crackles. 
 B. Rhonchi 
 C. High pitched wheeze 
 D. Stridor A young adult client is admitted to the emergency room following a motor vehicle collision. The client’s head hit the dashboard. Admission assessment include: Blood pressure 85/45 mm Hg, temperature 98.6 F, pulse 124 beat/minute and respirations 22 breath/minute. Based on these data, the nurse formulates the first portion of nursing diagnosis as “Risk of injury” What term best expresses the “related to” portion of nursing diagnosis? A. infection B. increase intracranial pressure C. shock D. head injury The healthcare provider prescribes the antibiotic Cefdinir (cephalosporin) 300mg PO every 12 h for a client with postoperative wound infections. Which feeds should the nurse encourage this client to eat? A. Yogurt and/or buttermilk. B. Avocados and cheese C. Green leafy vegetables D. Fresh fruits The charge nurse is making assignment on a psychiatric unit for a practical nurse (PN) and newly license register nurse (RN). Which client should be assigned to the RN? A. An adult female who has been depress for the past several month and denies suicidal ideations. B. A middle-age male who is in depressive phase on bipolar disease and is receiving Lithium. C. A young male with schizophrenia who said voices is telling him to kill his D. An elderly male who tell the staff and other client that he is superman and he can fly. The public nurse health received funding to initiate primary prevention program in the community. Which program the best fits the nurse’s proposal? • Case management and screening for clients with HIV.HESI • Regional relocation center for earthquake victims • Vitamin supplements for high-risk pregnant women. • Lead screening for children in low-income housing. When assessing and adult male who presents as the community health clinic with a history of hypertension, the nurse note that he has 2+ pitting edema in both ankles. He also has a history of gastroesophageal reflex disease (GERD) and depression. Which intervention is the most important for the nurse to implement? • Arrange to transport the client to the hospital • Instruct the client to keep a food journal, including portions size. • Review the client’s use of over the counter (OTC) medications. • Reinforce the importance of keeping the feet elevated. An older client is admitted to the intensive care unit with severe abdominal pain, abdominal distention, and absent bowel sound. The client has a history of smoking 2 packs of cigarettes daily for 50 years and is currently restless and confused. Vital signs are: temperature 96`F, heart rate 122 beats/minute, respiratory rate 36 breaths/minute, mean arterial pressure(MAP) 64 mmHg and central venous pressure (CVP) 7 mmHg. Serum laboratory findings include: hemoglobin 6.5 grams/dl, platelets 6o, 000, and white blood cell count (WBC) 3,000/mm3. Based on these findings this client is at greatest risk for which pathophysiological condition? • Multiple organ dysfunction syndrome (MODS) • Disseminated intravascular coagulation (DIC) • Chronic obstructive disease. • Acquired immunodeficiency syndrome (AIDS) A client experiencing withdrawal from the benzodiazepines alprazolam (Xanax) is demonstrating severe agitation and tremors. What is the best initial nursing action? • Administer naloxone (Narcan) per PNR protocol • Initiate seizure precautions • Obtain a serum drug screen • Instruct the family about withdrawal symptoms. The nurse is caring for a client who is taking a macrolide to treat a bacterial infection. Which finding should the nurse report to the healthcare provider before administering the next dose? • Jaundice • Nausea • Fever • Fatigue A client with Alzheimer’s disease (AD) is receiving trazodone (Desyrel), a recently prescribed atypical antidepressant. The caregiver tells the home health nurse that the client’s mood and sleep patterns are improved, but there is no change in cognitive ability. How should the nurse respond to this information? • Explain that it may take several weeks for the medication to be effective • Confirm the desired effect of the medication has been achieved. • Notify the health care provider than a change may be needed. • Evaluate when and how the medication is being administered to the client A client with diabetic peripheral neuropathy has been taking pregabalin (Lyrica) for 4 days. Which finding indicates to the nurse that the medication is effective? • Reduced level of pain • Full volume of pedal pulses • Granulating tissue in foot ulcer • Improved visual acuity A group of nurse-managers is asked to engage in a needs assessment for a piece of equipment that will be expensed to the organization’s budget. Which question is most important to consider when analyzing the cost-benefit for this piece of equipment? • How many departments can use this equipment? • Will the equipment require annual repair? • Is the cost of the equipment reasonable? • Can the equipment be updated each year? While receiving a male postoperative client’s staples de nurse observe that the client’s eyes are closed and his face and hands are clenched. The client states, “I just hate having staples removed”. After acknowledgement the client’s anxiety, what action should the nurse implement? • Encourage the client to continue verbalize his anxiety • Attempt to distract the client with general conversation • Explain the procedure in detail while removing the staples • Reassure the client that this is a simple nursing procedure. A male client is admitted for the removal of an internal fixation that was inserted for the fracture ankle. During the admission history, he tells the nurse he recently received vancomycin (vancomycin) for a methicillin-resistant Staphylococcus aureus (MRSA) wound infection. Which action should the nurse take? (Select all that apply.) • Collect multiple site screening culture for MRSA • Call healthcare provider for a prescription for linezolid (Zyrovix) • Place the client on contact transmission precautions • Obtain sputum specimen for culture and sensitivity • Continue to monitor for client sign of infection. The nurse plans to collect a 24- hour urine specimen for a creatinine clearance test. Which instruction should the nurse provide to the adult male client? • Clearance around the meatus, discard first portion of voiding, and collect the rest in a sterile bottle • Urinate at specific time, discard the urine, and collect all subsequent urine during the next 24 hours. • For the next 24 hours, notify the nurse when the bladder is full, and the nurse will collect catheterized specimens. • Urinate immediately into a urinal, and the lab will collect specimen every 6 hours, for the next 24 hours. The nurse assesses a client with new onset diarrhea. It is most important for the nurse to question the client about recent use of which type of medication? • Antibiotics • Anticoagulants • Antihypertensive • Anticholinergics A neonate with a congenital heart defect (CHD) is demonstrating symptoms of heart failure (HF). Which interventions should the nurse include in the infant’s plan of care? • Give O2 at 6 L/nasal cannula for 3 repeated oximetry screens below 90% • Administer diuretics via secondary infusion in the morning only • Evaluate heart rate for effectiveness of cardio tonic medications • Use high energy formula 30 calories/ounce at Q3 hours feeding via soft nipples • Ensure Interrupted and frequent rest periods between procedures. An elderly male client is admitted to the mental health unit with a sudden onset of global disorientation and is continuously conversing with his mother, who died 50 years ago. The nurse reviews the multiple prescriptions he is currently taking and assesses his urine specimen, which is cloudy, dark yellow, and has foul odor. These findings suggest that his client is experiencing which condition? • Delirium • Depression • Dementia • Psychotic episode Following an esophagogastroduodenoscopy (EGD) a male client is drowsy and difficult to arouse, and his respiration are slow and shallow. Which action should the nurse implement? Select all that apply. • Prepare medication reversal agent • Check oxygen saturation level • Apply oxygen via nasal cannula • Initiate bag- valve mask ventilation. • Begin cardiopulmonary resuscitation The nurse is planning preoperative teaching plan of a 12-years old child who is scheduled for surgery. To help reduce the child anxiety, which action is the best for the nurse to implement? • Give the child syringes or hospital mask to play it at home prior to hospitalization. • Include the child in pay therapy with children who are hospitalized for similar surgery. • Provide a family tour of the preoperative unit one week before the surgery is scheduled • Provide doll an equipment to re-enact feeling associated with painful procedures. When development a teaching plan for a client newly diagnosed type 1 diabetes, the nurse should explain that an increase thirst is an early sing of diabetes ketoacidosis (DKA), which action should the nurse instruct the client to implement if this sign of DKA occur? • Resume normal physical activity • Drink electrolyte fluid replacement • Give a dose of regular insulin per sliding scale • Measure urinary output over 24 hours. The nurse is teaching a group of clients with rheumatoid arthritis about the need to modify daily activities. Which goal should the nurse emphasize? • Protect joint function • Improve circulation • Control tremors • Increase weight bearing An adult client experiences a gasoline tank fire when riding a motorcycle and is admitted to the emergency department (ED) with full thickness burns to all surfaces of both lower extremities. What percentage of body surface area should the nurse document in the electronic medical record (EMR)? • 9 % • 18 % • 36 % • 45 % A client with hyperthyroidism is receiving propranolol (Inderal). Which finding indicates that the medication is having the desired effect? • Decrease in serum T4 levels • Increase in blood pressure • Decrease in pulse rate • Goiter no longer palpable An older male client with type 2 diabetes mellitus reports that has experiences legs pain when walking short distances, and that the pain is relieved by rest. Which client behavior indicates an understanding of healthcare teaching to promote more effective arterial circulation? • Consistently applies TED hose before getting dressed in the morning. • Frequently elevated legs thorough the day. • Inspect the leg frequently for any irritation or skin breakdown • Completely stop cigarette/ cigar smoking. The nurse performs a prescribed neurological check at the beginning of the shift on a client who was admitted to the hospital with a subarachnoid brain attack (stroke). The client’s Glasgow Coma Scale (GCS) score is 9. What information is most important for the nurse to determine? The client’s previous GCS score • When the client’s stroke symptoms started • If the client is oriented to time • The client’s blood pressure and respiration rate Based on principles of asepsis, the nurse should consider which circumstance to be sterile? • One inch- border around the edge of the sterile field set up in the operating room • A wrapped unopened, sterile 4x4 gauze placed on a damp table top. • An open sterile Foley catheter kit set up on a table at the nurse waist level • Sterile syringe is placed on sterile area as the nurse riches over the sterile field. An unlicensed assistive personnel (UAP) reports that a client’s right hand and fingers spasms when taking the blood pressure using the same arm. After confirming the presence of spams what action should the nurse take? • Ask the UAP to take the blood pressure in the other arm • Tell the UAP to use a different sphygmomanometer. • Review the client’s serum calcium level • Administer PRN antianxiety medication. A 56-years-old man shares with the nurse that he is having difficulty making decision about terminating life support for his wife. What is the best initial action by the nurse? • Provide an opportunity for him to clarify his values related to the decision • Encourage him to share memories about his life with his wife and family • Advise him to seek several opinions before making decision • Offer to contact the hospital chaplain or social worker to offer support. A client is being discharged home after being treated for heart failure (HF). What instruction should the nurse include in this client’s discharge teaching plan? • Weigh every morning • Eat a high protein diet • Perform range of motion exercises • Limit fluid intake to 1,500 ml daily A child with heart failure is receiving the diuretic furosemide (Lasix) and has serum potassium level 3.0 mEq/L. Which assessment is most important for the nurse to obtain? • Cardiac rhythm and heart rate. • Daily intake of foods rich in potassium. • Hourly urinary output • Thirst ad skin turgor. The nurse note a depressed female client has been more withdrawn and non-communicative during the past two weeks. Which intervention is most important to include in the updated plan of care for this client? • Encourage the client’s family to visit more often • Schedule a daily conference with the social worker • Encourage the client to participate in group activities A client with rheumatoid arthritis (RA) starts a new prescription of etanercept (Enbrel) subcutaneously once weekly. The nurse should emphasize the importance of reporting problem to the healthcare provider? • Headache • Joint stiffness • Persistent fever • Increase hunger and thirst • Engage the client in a non-threatening conversation. The nurse is assessing an older adult with type 2 diabetes mellitus. Which assessment finding indicates that the client understands long- term control of diabetes? • The fating blood sugar was 120 mg/dl this morning. • Urine ketones have been negative for the past 6 months • The hemoglobin A1C was 6.5g/100 ml last week • No diabetic ketoacidosis has occurred in 6 months. A 13 years-old client with non-union of a comminuted fracture of the tibia is admitted with osteomyelitis. The healthcare provider collects home aspirate specimens for culture and sensitivity and applies a cast to the adolescent’s lower leg. What action should the nurse implement next? • Administer antiemetic agents • Bivalve the cast for distal compromise • Provide high- calorie, high-protein diet • Begin parenteral antibiotic therapy A male client receives a thrombolytic medication following a myocardial infarction. When the client has a bowel movement, what action should the nurse implement? • Send stool sample to the lab for a guaiac test • Observe stool for a day-colored appearance. • Obtain specimen for culture and sensitivity analysis • Asses for fatty yellow streaks in the client’s stool. In early septic shock states, what is the primary cause of hypotension? • Peripheral vasoconstriction • Peripheral vasodilation • Cardiac failure • A vagal response A client diagnosed with calcium kidney stones has a history of gout. A new prescription for aluminum hydroxide (Amphogel) is scheduled to begin at 0730. Which client medication should the nurse bring to the healthcare provider’s attention? • Allopurinol (Zyloprim) • Aspirin, low dose • Furosemide (lasix) • Enalapril (vasote) A male client’s laboratory results include a platelet count of 105,000/ mm3 Based on this finding the nurse should include which action in the client’s plan of care? • Cluster care to conserve energy • Initiate contact isolation • Encourage him to use an electric razor • Asses him for adventitious lung sounds A client is admitted to the hospital after experiencing a brain attack, commonly referred to as a stroke or cerebral vascular accident (CVA). The nurse should request a referral for speech therapy if the client exhibits which finding? • Abnormal responses for cranial nerves I and II • Persistent coughing while drinking • Unilateral facial drooping • Inappropriate or exaggerated mood swings At 1615, prior to ambulating a postoperative client for the first time, the nurse reviews the client’s medical record. Based on date contained in the record, what action should the nurse take before assisting the client with ambulation: • Remove sequential compression devices. • Apply PRN oxygen per nasal cannula. • Administer a PRN dose of an antipyretic. • Reinforce the surgical wound dressing Which assessment finding for a client who is experiencing pontine myelinolysis should the nurse report to the healthcare provider? • Sudden dysphagia • Blurred visual field • Gradual weakness • Profuse diarrhea A client is scheduled to receive an IW dose of ondansetron (Zofran) eight hours after receiving chemotherapy. The client has saline lock and is sleeping quietly without any restlessness. The nurse caring for the client is not certified in chemotherapy administration. What action should the nurse take? • Ask a chemotherapy-certified nurse to administer the Zofran • Administer the Zofran after flushing the saline lock with saline • Hold the scheduled dose of Zofran until the client awakens • Awaken the client to assess the need for administration of the Zofran. When providing diet teaching for a client with cholecystitis, which types of food choices the nurse recommend to the client? • High protein • Low fat • Low sodium • High carbohydrate. A client with a history of cirrhosis and alcoholism is admitted with severe dyspnea and ascites. Which assessment finding warrants immediate intervention by the nurse? • Jaundice skin tone • Muffled heart sounds • Pitting peripheral edema • Bilateral scleral edema When entering a client’s room, the nurse discovers that the client is unresponsive and pulseless. The nurse initiate CPR and Calls for assistance. Which action should the nurse take next? • Prepare to administer atropine 0.4 mg IVP • Gather emergency tracheostomy equipment • Prepare to administer lidocaine at 100 mg IVP • Place cardiac monitor leads on the client’s chest. A client with a history of dementia has become increasingly confused at night and is picking at an abdominal surgical dressing and the tape securing the intravenous (IV) line. The abdominal dressing is no longer occlusive, and the IV insertion site is pink. What intervention should the nurse implement? • Replace the IV site with a smaller gauge. • Redress the abdominal incision • Leave the lights on in the room at night. • Apply soft bilateral wrist restraints. An adult male client is admitted to the emergency room following an automobile collision in which he sustained a head injury. What assessment data would provide the earliest that the client is experiencing increased intracranial pressure (ICP)? • Lethargy • Decorticate posturing • Fixed dilated pupil • Clear drainage from the ear. In preparing a diabetes education program, which goal should the nurse identify as the primary emphasis for a class on diabetes self-management? • Prepare the client to independently treat their disease process • Reduce healthcare costs related to diabetic complications • Enable clients to become active participating in controlling the disease process • Increase client’s knowledge of the diabetic disease process and treatment options To reduce staff nurse role ambiguity, which strategy should the nurse manager implemented? • Confirm that all the staff nurses are being assigned to equal number of clients. • Review the staff nurse job description to ensure that it is clear, accurate, and recurrent. • Assign each staff nurse a turn unit charge nurse on a regular, rotating basis. • Analyze the amount of overtime needed by the nursing staff to complete assignments. The nurse is assisting a new mother with infant feeding. Which information should the nurse provide that is most likely to result in a decrease milk supply for the mother who is breastfeeding? • Supplemental feedings with formula • Maternal diet high in protein • Maternal intake of increased oral fluid • Breastfeeding every 2 or 3 hours. Which assessment is more important for the nurse to include in the daily plan of care for a client with a burned extremity? • Range of Motion • Distal pulse intensity • Extremity sensation • Presence of exudate When assessing a 6-month old infant, the nurse determines that the anterior fontanel is bulging. In which situation would this finding be most significant? • Crying • Straining on stool • Vomiting • Sitting upright. A client with angina pectoris is being discharge from the hospital. What instruction should the nurse plan to include in this discharge teaching? • Engage in physical exercise immediately after eating to help decrease cholesterol levels. • Walk briskly in cold weather to increase cardiac output • Keep nitroglycerin in a light-colored plastic bottle and readily available. • Avoid all isometric exercises, but walk regularly. What is the priority nursing action when initiating morphine therapy via an intravenous patient-controlled analgesia (PCA) pump? • Initiate the dosage lockout mechanism on the PCA pump • Instruct the client to use the medication before the pain becomes severe • Assess the abdomen for bowel sounds. • Assess the client ability to use a numeric pain scale The nurse receives a newborn within the first minutes after a vaginal delivery and intervenes to establish adequate respirations. What priority issue should the nurse address to ensure the newborn’s survival? • Heat loss • Hypoglycemia • Fluid balance • Bleeding tendencies A male client with a long history of alcoholism is admitted because of mild confusion and fine motor tremors. He reports that he quit drinking alcohol and stopped smoking cigarettes one month ago after his brother died of lung cancer. Which intervention is most important for the nurses to include in the client’s plan of care? • Determine client’s level current blood alcohol level. • Observe for changes in level of consciousness. • Involve the client’s family in healthcare decisions. • Provide grief counseling for client and his family. An older adult female admitted to the intensive care unit (ICU) with a possible stroke is intubated with ventilator setting of tidal volume 600, PlO2 40%, and respiratory rate of 12 breaths/minute. The arterial blood gas (ABG) results after intubation are PH 7.31. PaCO2 60, PaO2 104, SPO2 98%, HCO3 23. To normalize the client’s ABG finding, which action is required? • Report the results to the healthcare provider. • Increase ventilator rate. • Administer a dose of sodium carbonate. • Decrease the flow rate of oxygen. The mother of the 12- month-old with cystic fibrosis reports that her child is experiencing increasing congestion despite the use of chest physical therapy (CPT) twice a day, and has also experiences a loss of appetite. What instruction should the nurse provide? • Perform CPT after meals to increase appetite and improve food intake. • CPT should be performed more frequently, but at least an hour before meals. • Stop using CPT during the daytime until the child has regained an appetite. • Perform CPT only in the morning, but increase frequency when appetite improves. A client with type 2 diabetes mellitus is admitted for frequent hyperglycemic episodes and a glycosylated hemoglobin (HbA1c) of 10%. Insulin glargine 10 units subcutaneously once a day at bedtime and a sliding scale with insulin aspart q6h are prescribed. What action should the nurse include in this client’s plan of care? • Fingerstick glucose assessment q6h with meals • Mix bedtime dose of insulin glargine with insulin aspart sliding scale dose • Review with the client proper foot care and prevention of injury • Do not contaminate the insulin aspart so that it is available for iv use • Coordinate carbohydrate controlled meals at consistent times and intervals • Teach subcutaneous injection technique, site rotation and insulin management Which problem reported by a client taking lovastatin requires the most immediate fallow up by the nurse? • Diarrhea and flatulence • Abdominal cramps • Muscle pain • Altered taste While assessing a client’s chest tube (CT), the nurse discovers bubbling in the water seal chamber of the chest tube collection device. The client’s vital signs are: blood pressure of 80/40 mmHg, heart rate 120 beats/minutes, respiratory rate 32 breaths/minutes, oxygen saturation 88%. Which interventions should the nurse implement? • Provide supplemental oxygen • Auscultate bilateral lung fields • Administer a nebulizer treatment • Reinforce occlusive CT dressing • Give PRN dose of pain medication Before leaving the room of a confused client, the nurse notes that a half bow knot was used to attach the client's wrist restraints to the movable portion of the client's bed frame. What action should the nurse take before leaving the room? • Ensure that the knot can be quickly released. • Tie the knot with a double turn or square knot. • Move the ties so the restraints are secured to the side rails. • Ensure that the restraints are snug against the client's wrist. Oral antibiotics are prescribed for an 18-month-old toddler with severe otitis media. An antipyrine and benzocaine-otic also prescribed for pain and inflammation. What instruction should the nurse emphasize concerning the installation of the antipyrine/benzocaine otic solution? • Place the dropper on the upper outer ear canal and instill the medication slowly. • Warm the medication in the microwave for 10 seconds before instilling. • Keep the medication refrigerated between administrations. • Have the child lie with the ear up for one to two minute after installation. An older adult male is admitted with complications related to chronic obstructive pulmonary disease (COPD). He reports progressive dyspnea that worsens on exertion and his weakness has increased over the past month. The nurse notes that he has dependent edema in both lower legs. Based on these assessment findings, which dietary instruction should the nurse provide? • Limit the intake of high calorie foods. • Eat meals at the same time daily. • Maintain a low protein diet. • Restrict daily fluid intake. The nurse inserts an indwelling urinary catheter as seen in the video what action should the nurse take next? • Remove the catheter and insert into urethral opening • Observe for urine flow and then inflate the balloon. • Insert the catheter further and observe for discomfort. • Leave the catheter in place and obtain a sterile catheter. A client with coronary artery disease who is experiencing syncopal episodes is admitted for an electrophysiology study (EPS) and possible cardiac ablation therapy. Which intervention should the nurse delegate to the unlicensed assistive personnel (UAP)? • Prepare the skin for procedure. • Identify client's pulse points • Witness consent for procedure • Check telemetry monitoring Fallowing an outbreak of measles involving 5 students in an elementary school, which action is most important for the school nurse to take? • Review the immunization records of all children in the elementary school • Report the measles outbreak to all community health organizations • Schedule a mobile public health vehicle to offer measles inoculations to unvaccinated children. • Restrict unvaccinated children from attending school until measles outbreak is resolved. A preeclamptic client who delivered 24h ago remains in the labor and delivery recovery room. She continues to receive magnesium sulfate at 2 grams per hour. Her total input is limited to 125 ml per hour, and her urinary output for the last hour was 850 ml. What intervention should the nurse implement? • discontinue the magnesium sulfate immediately • Decrease the client's iv rate to 50 ml per hour • Continue with the plan of care for this client • Change the client's to NPO status The nurse is planning care for a client who admits having suicidal thoughts. Which client behavior indicates the highest risk for the client acting on these suicidal thoughts? • Express feelings of sadness and loneliness • Neglects personal hygiene and has no appetite • Lacks interest in the activity of the family and friends • Begin to show signs of improvement in affect When assessing a multigravida the first postpartum day, the nurse finds a moderate amount of lochia rubra, with the uterus firm, and three fingerbreadths above the umbilicus. What action should the nurse implement first? • Massage the uterus to decrease atony • Check for a destined bladder • Increase intravenous infusion • Review the hemoglobin to determined hemorrhage A 12 year old client who had an appendectomy two days ago is receiving 0.9% normal saline at 50 ml/hour. The client's urine specific gravity is 1.035. What action should the nurse implement? • Evaluate postural blood pressure measurements • Obtain specimen for uranalysis • Encourage popsicles and fluids of choice • Assess bowel sounds in all quadrants An older male client arrives at the clinic complaining that his bladder always feels full. He complains of weak urine flow, frequent dribbling after voiding, and increasing nocturia with difficulty initiating his urine stream. Which action should the nurse implement? • Obtain a urine specimen for culture and sensitivity • Palpate the client's suprapubic area for distention • Advise the client to maintain a voiding diary for one week • Instruct in effective technique to cleanse the glans penis The nurse is preparing to administer 1.6 ml of medication IM to a 4 month old infant. Which action should the nurse include? • Select a 22 gauge 1 ½ inch (3.8 cm) needle for the intramuscular injection • Administer into the deltoid muscle while the parent holds the infant securely • Divide the medication into two injection with volumes under 1ml • Use a quick dart-like motion to inject into the dorsogluteal site. A client who had a below the knee amputation is experiencing severe phantom limb pain (PLP) and ask the nurse if mirror therapy will make the pain stop. Which response by the nurse is likely to be most helpful? • Research indicates that mirror therapy is effective in reducing phantom limb pain • You can try mirror therapy, but do not expect to complete elimination of the pain • Transcutaneous electrical nerve stimulators (TENS) have been found to be more effective • Where did you learn about the use of mirror therapy in treating in treating phantom limb pain? An older adult client with heart failure (HF) develops cardiac tamponade. The client has muffled, distant, heart sounds, and is anxious and restless. After initiating oxygen therapy and IV hydration, which intervention is most important for the nurse to implement? • Observe neck for jugular vein distention • Notify healthcare provider to prepare for pericardiocentesis • Asses for paradoxical blood pressure • Monitor oxygen saturation (Sp02) via continuous pulse oximetry A new member joins the nursing team spreads books on the table, puts items on two chairs, and sits on a third chair. The members of the group are forced to move closer and remove their possessions from the table what action should the nurse leader take? • Move to welcome and accommodate a new person • Ask the new person to move belonging to accommodate others • Tell the new person to move belongings because of limited space • Bring in additional chairs so that all staff members can be seated The nurse is caring for a one week old infant who has a ventriculoperitoneal (VP) shunt that was placed 2 days after birth. Which findings are an indication of a postoperative complication? • Poor feeding and vomiting • Leakage of CSF from the incisional site • Hyperactive bowel sound • Abdominal distention • WBC count of 10000/mm3 In monitoring tissue perfusion in a client following an above the knee amputation (aka), which action should the nurse include in the plan of care? • Evaluate closet proximal pulse. • Asses skin elasticity of the stump. • Observe for swelling around the stump. • Note amount color of wound drainage. The leg of a client who is receiving hospice care have become mottled in appearance. When the nurse observes the unlicensed assistive personal (UAP) place a heating pad on the mottled areas, what action should the nurse take? • Remove the heating pads and place a soft blanket over the client’s leg and feet. • Advise the UAP to observe the client’s skin while the heating pads are in place. • Elevate the client’s feet on a pillow and monitor the client’s pedal pulses frequently. • Instruct the UAP to reposition the heating pads to the sides of the legs and feet. A client who underwent an uncomplicated gastric bypass surgery is having difficult with diet management. What dietary instruction is most important for the nurse to explain to the client? • Chew food slowly and thoroughly before attempting to swallow • Plan volume-controlled evenly-space meal thorough the day • Sip fluid slowly with each meal and between meals • Eliminate or reduce intake fatty and gas forming food If the nurse is initiating IV fluid replacement for a child who has dry, sticky mucous membranes, flushed skin, and fever of 103.6 F. Laboratory finding indicate that the child has a sodium concentration of 156 mEq/L. What physiologic mechanism contributes to this finding? • The intravenous fluid replacement contains a hypertonic solution of sodium chloride • Urinary and Gastrointestinal fluid loss reduce blood viscosity and stimulate thirst • Insensible loss of body fluids contributes to the hemoconcentration of serum solutes • Hypothalamic resetting of core body temperature causes vasodilation to reduce body heat During a Woman’s Health fair, which assignment is the best for the Practical Nurse (PN) who is working with a register nurse (RN) • Encourage the woman at risk for cancer to obtain colonoscopy. • Present a class of breast-self examination • Prepare a woman for a bone density screening • Explain the follow-up need it for a client with prehypertension. An adult client present to the clinic with large draining ulcers on both lower legs that are characteristics of Kaposi’s sarcoma lesions. The client is accompanied by two family member. Which action should the nurse take? • Ask family member to wear gloves when touching the patient • Send family to the waiting area while the client’s history is taking • Obtain a blood sample to determine is the client is HIV positive • Complete the head to toes assessment to identify other sign of HIV An adult client is exhibit the maniac stage of bipolar disorder is admitted to the psychiatric unit. The client has lost 10 pounds in the last two weeks and has no bathed in a week “I’m trying to start a new business and “I’m too busy to eat”. The client is oriented to time, place, person but not situation. Which nursing problem has the greatest priority? • Hygiene-self-care deficit • Imbalance nutrition • Disturbed sleep pattern • Self-neglect The nurse is preparing a discharge teaching plan for a client who had a liver transplant. Which instruction is most important to include in this plan? • Limit intake fatty foods for one month after surgery. • Notify the healthcare provider if edema occurs. • Increase activity and exercise gradually, as tolerated. • Avoid crowds for first two months after surgery. A client who had a percutaneous transluminal coronary angioplasty (PTCA) two weeks ago returns to the clinic for a follow up visit. The client has a postoperative ejection fraction ejection fraction of 30%. Today the client has lungs which are clear, +1 pedal edema, and a 5pound weight gain. Which intervention the nurse implement? • Arrange transport for admission to the hospital. • Insert saline lock for IV diuretic therapy. • Assess compliance with routine prescriptions. • Instruct the client to monitor daily caloric intake. The RN is assigned to care for four surgical clients. After receiving report, which client should the nurse see first? The client who is • Two days postoperative bladder surgery with continuous bladder irrigation infusing. • One day postoperative laparoscopic cholecystectomy requesting pain medication. • Three days postoperative colon resection receiving transfusion of packed RBCs. • Preoperative, in buck’s traction, and scheduled for hip arthroplasty within the next 12 hours. The nurse is preparing an older client for discharge following cataract extraction. Which instruction should be include in the discharge teaching? • Do not read without direct lighting for 6 weeks. • Avoid straining at stool, bending, or lifting heavy objects. • Irrigate conjunctiva with ophthalmic saline prior to installing antibiotic ointment. • Limit exposure to sunlight during the first 2 weeks when the cornea is healing. At 40 week gestation, a laboring client who is lying is a supine position tells the nurse that she has finally found a comfortable position. What action should the nurse take? • Encourage the client to turn on her left side. • Place a pillow under the client’s head and knees. • Explain to the client that her position is not safe. • Place a wedge under the client’s right hip. A client with a history of diabetes and coronary artery disease is admitted with shortness of breath, anxiety, and confusion. The client’s blood pressure is 80/60 mmHg, heart rate 120 beats/minute with audible third and fourth heart sounds, and bibasilar crackles. The client’s average urinary output is 5 ml/hour. Normal saline is infusing at 124 ml/hour with a secondary infusion of dopamine at mcg/kg/minute per infusion pump. With intervention should the nurse implement? • Irrigate the indwelling urinary catheter. • Prepare the client for external pacing. • Obtain capillary blood glucose measurement. • Titrate the dopamine infusion to raise the BP. The nurse ends the assessment of a client by performing a mental status exam. Which statement correctly describes the purpose of the mental status exam? • Determine the client’s level of emotional functioning’ • Assess functional ability of the primary support system. • Evaluate the client’s mood, cognition and orientation. • Review the client’s pattern of adaptive coping skill An older adult resident of a long-term care facility has a 5-year history of hypertension. The client has a headache and rate the pain 5 on a pain scale 0 to 10. The client’s blood pressure is currently 142/89. Which interventions should the nurse implement? (Select all that apply) • Administer a daily dose of lisinopril as scheduled. • Assess the client for postural hypotension. • Notify the healthcare provider immediately • Provide a PRN dose of acetaminophen for headache • Withhold the next scheduled daily dose of warfarin. When conducting diet teaching for a client who is on a postoperative soft diet, which foods should eat? (Select all that apply) • Pasta, noodles, rice. • Egg, tofu, ground meat. • Mashed, potatoes, pudding, milk. • Brussel sprouts, blackberries, seeds. • Corn bran, whole wheat bread, whole grains. The nurse is preparing a 4-day-old I infant with a serum bilirubin level of 19 mg/dl (325 micromol/L) for discharge from the hospital. When teaching the parents about home phototherapy, which instruction should the nurse include in the discharge teaching plan? • Reposition the infant every 2 hours. • Perform diaper changes under the light. • Feed the infant every 4 hours. • Cover with a receiving blanket. When planning care for a client with acute pancreatitis, which nursing intervention has the highest priority? • Withhold food and fluid intake. • Initiate IV fluid replacement. • Administer antiemetic as needed. • Evaluate intake and output ratio. Assessment by the home health nurse of an older client who lives alone indicates that client has chronic constipations. Daily medications include furosemide for hypertension and heart failure and laxatives. To manage the client’s constipation, which suggestions should the nurse provide? (Select all that apply) • Decrease laxative use to every other day, and use oil retention enemas as needed. • Include oatmeal with stewed pruned for breakfast as often as possible. • Increase fluid intake by keeping water glass next to recliner. • Recommend seeking help with regular shopping and meal preparation. • Report constipation to healthcare provider related to cardiac medication side effects. A young boy who is in a chronic vegetative state and living at home is readmitted to the hospital with pneumonia and pressure ulcers. The mother insists that she is capable of caring for her son and which action should the nurse implement next? • Report the incident to the local child protective services. • Find a home health agency that specializes in brain injuries. • Determine the mother’s basic skill level in providing care. • Consult the ethics committee to determine how to proceed. After the risk and benefits of having a cardiac catheterization are reviewed by the healthcare provider, an older adult with unstable angina is scheduled for the procedure. When the nurse presents the consent form for signature, the client asks how the wires will keep a heart heating during the procedure. What action should the nurse take? • Explain the procedure again in detail and clarify any misconceptions. • Notify the healthcare provider of the client’s lack of understanding. • Call the client’s next of kin and have them provide verbal consent. • Postpone the procedure until the client understands the risk and benefits. In assessing a client at 34-weeks’ gestation, the nurse notes that she has a slightly elevated total T4 with a slightly enlarged thyroid, a hematocrit of 28%, a heart rate of 92 beats per minute, and a systolic murmur. Which finding requires follow-up? • Elevated thyroid hormone level. • Hematocrit of 28%. • Heart rate of 92 beats per minute. • Systolic murmur. A client with osteoporosis related to long-term corticosteroid therapy receives a prescription for calcium carbonate. Which client’s serum laboratory values requires intervention by the nurse? • Total calcium 9 mg/dl (2.25 mmol/L SI) • Creatinine 4 mg/dl (354 micromol/L SI) • Phosphate 4 mg/dl (1.293 mmol/L SI) • Fasting glucose 95 mg/dl (5.3 mmol/L SI) A clinical trial is recommended for a client with metastatic breast cancer, but she refuses to participate and tells her family that she does not wish to have further treatments. The client’s son and daughter ask the nurse to try and convince their mother to reconsider this decision. How should the nurse respond? • Ask the client with her children present if she fully understands the decision she has made. • Discuss success of clinical trials and ask the client to consider participating for one month. • Explain to the family that they must accept their mother’s decision. • Explore the client’s decision to refuse treatment and offer support An adult client with severe depression was admitted to the psychiatric unit yesterday evening. Although the client ran one year ago, his spouse states that the client no longer runs, bur sits and watches television most of the day. Which is most important for the nurse to include in this client’s plan of care for today? • Assist client in identifying goals for the day. • Encourage client to participate for one hour in a team sport. • Schedule client for a group that focuses on self-esteem. • Help client to develop a list of daily affirmations. An adult who is 5 feet 5 inches (165.1 cm) tall and weighs 90 lb. (40.8 Kg) is admitted with a diagnosis of chronic anorexia. The client receives a regular diet for 2 days, and the client’s medical records indicates that 100% of the diet provided has been consumed. However the client’s weight on the third day morning after admission is 89 lb. (40.4 Kg). What action should the nurse implement? • Examine the client’s room for hidden food. • Assign staff to monitor what the client eats. • Ask the client if the food provided is being eaten or discarded. • Provide the client with a high calorie diet. A client exposed to tuberculosis is scheduled to begin prophylactic treatment with isoniazid. Which information is most important for the nurse to note before administering the initial dose? • Conversion of the client’s PPD test from negative to positive. • Length of time of the exposure to tuberculosis. • Current diagnosis of hepatitis B. • History of intravenous drug abuse. The nurse walks into a client’s room and notices bright red blood on the sheets and on the floor by the IV pole. Which action should the nurse take first? • Clean up the spilled blood to reduce infection transmission. • Notify the healthcare provider that the client appears to be bleeding. • Apply direct pressure to the client’s IV site. • Identify the source and amount of bleeding. During a routine clinic visit, an older female adult tells the nurse that she is concerned that the flu season is coming soon, but is reluctant to obtain the vaccination. What action should the nurse take first? • Determine when the client last had an influenza vaccination. • Discuss the concerns expressed by the client about the vaccination. • Ask about any recent exposure to persons with the flu or other viruses. • Review the informed consent form for the vaccination with the client. A client is admitted with acute pancreatitis. The client admits to drinking a pint of bourbon daily. The nurse medicates the client for pain and monitors vital signs q2 hours. Which finding should the nurse report immediately to the healthcare provider? • Confusion and tremors • Yellowing and itching of skin. • Abdominal pain and vomiting • Anorexia and abdominal distention The nurse is teaching a mother of a newborn with a cleft lip how to bottle feed her baby using medela haberman feeder, which has a valve to control the release of milk and a slit nipple opening. The nurse discusses placing the nipple’s elongated tip in the back of the oral cavity. What instructions should the nurse provide the mother about feedings? • Squeeze the nipple base to introduce milk into the mouth • Position the baby in the left lateral position after feeding • Alternate milk with water during feeding • Hold the newborn in an upright position Following and gunshot wound, an adult client a hemoglobin level of 4 grams/dl (40 mmol/L SI). The nurse prepares to administer a unit of blood for an emergency transfusion. The client has AB negative blood type and the blood bank sends a unit of type A Rh negative, reporting that there is not type AB negative blood currently available. Which intervention should the nurse implement? • Transfuse Type A negative blood until type AB negative is available. • Recheck the client’s hemoglobin, blood type and Rh factor. • Administer normal saline solution until type AB negative is available • Obtain additional consent for administration of type A negative blood A young adult female college student visits the health clinic in early winter to obtain birth control pills. The clinic nurse asks if the student has received an influenza vaccination. The student stated she did not receive vaccination because she has asthma. How should the nurse respond? • Offer to provide the influenza vaccination to the student while she is at the clinic • Encourage the student to obtain a vaccination prior to the next influenza season. • Confirm that a history of asthma can increase risks associated with the vaccine. • Advise the student that the nasal spray vaccine reduces side effects for people with asthma. A client with eczema is experiencing severe pruritus. Which PRN prescriptions should the nurse administer? (Select all that apply) • Topical corticosteroid. • Topical scabicide. • Topical alcohol rub. • Transdermal analgesic. • Oral antihistamine An adult male was diagnosed with stage IV lung cancer three weeks ago. His wife approaches the nurse and asks how she will know that her husband's death is imminent because their two adult children want to be there when he dies. What is the best response by the nurse? • Explain that the client will start to lose consciousness and his body system will slow down • Reassure the spouse that the healthcare provider will let her know when to call the children • Offer to discuss the client’s health status with each of the adult children • Gather information regarding how long it will take for the children to arrive When should intimate partner violence (IPV) screening occur? • As soon as the clinician suspects a problem • Only when a client presents with an unexplained injury • As a routine part of each healthcare encounter • Once the clinician confirms a history of abuse A child newly diagnosed with sickle cell anemia (SCA) is being discharged from the hospital. Which information is most important for the nurse to provide the parents prior to discharge? • Instructions about how much fluid the child should drink daily • information about non-pharmaceutical pain reliever measures • Referral for social services for the child and family • Signs of addiction to opioid and medications What action should the school nurse implement to provide secondary prevention to a school-age children? • Collaborate with a science teacher to prepare a health lesson • Prepare a presentation on how to prevent the spread of lice • Initiate a hearing and vision screening program for first-graders • Observe a person with type 1 diabetes self-administer a dose of insulin While assisting a client who recently had a hip replacement into a bed pan, the nurse notices that there is a small amount of bloody drainage on the surgical dressing, the client’s skin is warm to the touch, and there is a strong odor from the urine. Which action should the nurse take? • Obtain a urine sample from the bed pan • Remove dressing and assess surgical site • Insert an indwelling urinary catheter • Measure the client’s oral temperature While making rounds, the charge nurse notices that a young adult client with asthma who was admitted yesterday is sitting on the side of the bed and leaning over the bed-side-table. The client is currently receiving at 2 litters/minute via nasal cannula. The client is wheezing and is using pursed-lip breathing. Which intervention should the nurse implement? • Assist the client to lie back in bed • Call for an Ambu resuscitating bag • Increase oxygen to 6 litters/minute • Administer a nebulizer Treatment A client with emphysema is being discharged from the hospital. The nurse enters the client’s room to complete discharge teaching. The client reports feeling a little short of breath and is anxious about going home. What is the best course of action? • Postpone discharge instructions at this time and offer to contact the client by phone in a few days • Invite the client to return to the unit for discharge teaching in a few days, when there is less anxiety • Provide only necessary information in short, simple explanations with written instructions to take home • Give detailed instructions speaking slowly and clearly while looking directly at the client when speaking An older adult male who had an abdominal cholecystectomy has become increasingly confused and disoriented over the past 24 hours. He is found wandering into another client’s room and is return to his room by the unlicensed assistive personnel (UAP). What actions should the nurse take? (Select all that apply). • Apply soft upper limb restrains and raise all four bed rails • Report mental status change to the healthcare provider • Assess the client’s breath sounds and oxygen saturation • Assign the UAP to re-assess the client’s risk for falls • Review the client’s most recent serum electrolyte values A client is admitted to a medical unit with the diagnosis of gastritis and chronic heavy alcohol abuse. What should the nurse administered to prevent the development of Wernicke's syndrome? • Lorazepam (Ativan) • Famotidine (Pepcid) • Thiamine (Vitamin B1) • Atenolol (Tenormin) When conducting diet teaching for a client who was diagnosed with nutritional anemia in pregnancy, which foods should the nurse encourage the client to eat? (Select all that apply) • Seeds, spices, lettuce • Consomme, celery, carrot • Oranges, orange juice, bananas • Fortified whole wheat cereals, whole-grain pasta, brown rice • Spinach, kale, dried raisins and apricots A client with type 2 diabetes mellitus is admitted for antibiotic treatment for a leg ulcer. To monitor the client for the onset of hyperosmolar hyperglycemic nonketotic syndrome (HHNS), what actions should the nurse take? (Select all that apply) • Check urine for ketones • Measure blood glucose • Monitor vital signs • Assessed level of consciousness After receiving report, the nurse can most safely plan to assess which client last? The client with… • A rectal tube draining clear, pale red liquid drainage • A distended abdomen and no drainage from the nasogastric tube • No postoperative drainage in the Jackson-Pratt drain with the bulb compressed • Dark red drainage on a postoperative dressing, but no drainage in the Hemovac®. The nurse instructs an unlicensed assistive personnel (UAP) to turn an immobilized elderly client with an indwelling urinary catheter every two hours. What additional action should the nurse instruct the UAP to take each time the client is turned? • Empty the urinary drainage bag • Feed the client a snack • Offer the client oral fluids • Assess the breath sounds The nurse is preparing a client who had a below-the-knee (BKA) amputation for discharge to home. Which recommendations should the nurse provide this client? (Select all that apply) • Inspect skin for redness • Use a residual limb shrinker • Apply alcohol to the stump after bathing • Wash the stump with soap and water • Avoid range of motion exercises When assessing the surgical dressing of a client who had abdominal surgery the previous day, the nurse observes that a small amount of drainage is present on the dressing and the wound’s Hemovac suction device is empty with the plug open. How should the nurse respond? • Replace the dressing and remove the drainage device • Reposition the drainage device and keep the plug open • Notify the healthcare provider that the drain is not working • Recompress the wound suction device and secure to plug A mother brings her 4-month-old son to the clinic with a quarter taped over his umbilicus, and tells the nurse the quarter is supposed to fix her child’s hernia. Which explanations should the nurse provide? • This hernia is a normal variation that resolves without treatment. • Restrictive clothing will be adequate to help the hernia go away. • An abdominal binder can be worn daily to reduce the protrusion. • The quarter should be secured with an elastic bandage wrap. A client who is admitted to the intensive care unit with syndrome of inappropriate antidiuretic hormone (SIADH) has developed osmotic demyelination. Which intervention should the nurse implement first? • Patch one eye. • Reorient often. • Range of motion. • Evaluate swallow A client with possible acute kidney injury (AKI) is admitted to the hospital and mannitol is prescribed as a fluid challenge. Prior to carrying out this prescription, what intervention should the nurse implement? • Collect a clean catch urine specimen. • Instruct the client to empty the bladder. • Obtain vital signs and breath sounds. • No specific nursing action is required A male client with COPD smokes two packs of cigarettes per day and is admitted to the hospital for a respiratory infection. He complains that he has trouble controlling respiratory distress at home when using his rescue inhaler. Which comment from the client indicates to the nurse that he is not using his inhaler properly? • “I have a hard time inhaling and holding my breath after I squeeze the inhaler, but I do my best” • “ I never use the inhaler unless I am feeling really short of breath” • I always shake the inhaler several times before I start” • “After I squeeze the inhaler and swallow, I always feel a slight wave of nausea, bit it goes away” A 6 -years-old who has asthma is demonstrating a prolonged expiratory phase and wheezing, and has 35% personal best peak expiratory flow rate (PEFR). Based on these finding, which action should the nurse implement first? • Administer a prescribed bronchodilator. • Report finding to the healthcare provider. • Encourage the child to cough and deep breath • Determine what trigger precipitated this attack. A client is receiving lactulose (Portalac) for signs of hepatic encephalopathy. To evaluate the client’s therapeutic response to this medication, which assessment should the nurse obtain? • Level of consciousness • Percussion of abdomen • Serum electrolytes • Blood glucose. When administering an immunization in an adult client, the nurse palpates and administer the injection one inch below the acromion process into the center of the muscle mass. The nurse should document that the vaccine was administered at what site? • Rectus femenis • Ventrogluteous • Vastus lateralis • Deltoid In making client care assignment, which client is best to assign to the practical nurse (PN) working on the unit with the nurse? • An immobile client receiving low molecular weight heparin q12 h. • A client who is receiving a continuous infusion of heparin and gets out of bed BID • A client who is being titrated off heparin infusion and started on PO warfarin (Coumadin) • An ambulatory client receiving warfarin (Coumadin) with INR of 5 second. A client who is admitted to the intensive care unit with a right chest tube attached to a THORA-SEAL chest drainage unit becomes increasingly anxious and complain of difficulty breathing. The nurse determine the client is tachypneic with absent breath sounds in the client’s right lungs fields. Which additional finding indicates that the client has developed a tension pneumothorax? • Continuous bubbling in the water seal chamber • Decrease bright red blood drainage • Tachypnea and difficulty breathing • Tracheal deviation toward the left lung. A low-risk primigravida at 28-weeks gestation arrives for her regular antepartal clinic visit. Which assessment finding should the nurse consider within normal limits for this client? • Pulse increase of 10 beats/minute • Proteinuria • Glucosuria • Fundal height 0f 22 centimeters A client with chronic alcoholism is admitted with a decreased serum magnesium level. Which snack option should the nurse recommend to this client? • Cheddar cheese and crackers. • Carrot and celery sticks. • Beef bologna sausage slices. • Dry roasted almonds. The nurse is preparing a teaching plan for an older female client diagnosed with osteoporosis. What expected outcome has the highest priority for this client? • Identifies 2 treatments for constipation due to immobility. • Names 3 home safety hazards to be resolve immediately. • State 4 risk factors for the development of osteoporosis. • Lists 5 calcium-rich foods to be added to her daily diet. The nurse is teaching a male adolescent recently diagnosed with type 1diabetes mellitus (DM) about self-injecting insulin. Which approach is best for the nurse to use to evaluate do you effectiveness of the teaching? • Ask the adolescent to describe his level of comfort with injecting himself with insulin. • Observe him as he demonstrates self-injection technique in another diabetic adolescent • Have the adolescent list the procedural steps for safe insulin administration. • Review his glycosylated hemoglobin level 3 months after the teaching session. A young adult woman visits the clinic and learns that she is positive for BRCA1 gene mutation and asks the nurse what to expect next. How should the nurse respond? • Explain that counseling will be provided to give her information about her cancer risk • Gather additional information about the client’s family history for all types of cancer. • Offer assurance that there are a variety of effective treatments for breast cancer. • Provide information about survival rates for women who have this genetic mutation. A mother runs into the emergency department with s toddler in her arms and tells the nurse that her child got into some cleaning products. The child smells of chemicals on hands, face, and on the front of the child's clothes. After ensuring the airway is patent, what action should the nurse implement first? • Call poison control emergency number. • Determine type of chemical exposure. • Obtain equipment for gastric lavage. • Assess child for altered sensorium. The nurse assigned unlicensed assistive personnel (UAP) to apply antiembolism stockings to a client. The nurse and UAP enters the room, the nurse observes the stockings that were applying by the UAP. The UAP states that the client requested application of the stockings as seen on the picture, for increased comfort. What action should the nurse take? • Ask the client if the stocking feel comfortable. • Supervise the UAP in the removal of the stockings. • Place a cover over the client’s toes to keep them warm. • Discussed effective use of the stockings with the client on UAP While changing a client’s chest tube dressing, the nurse notes a crackling sensation when gentle pressure is applied to the skin at the insertion site. What is the best action for the nurse to take? • Apply a pressure dressing around the chest tube insertion site. • Assess the client for allergies to topical cleaning agents. • Measure the area of swelling and crackling. • Administer an oral antihistamine per PRN protocol. To prevent infection by auto contamination during the acute phase of recovery from multiple burns, which intervention is most important for the nurse to implement? • Dress each wound separately. • Avoid sharing equipment between multiple clients. • Use gown, mask and gloves with dressing change. • Implement protective isolation. The nurse is preparing an intravenous (IV) fluid infusion using an IV pump. Within 30 seconds of turning on the machine, the pump’s alarm beeps “occlusion”. What action should the nurse implement first? • Flush the vein with 3 ml of sterile normal saline. • Assess the IV catheter insertion site for infiltration. • Verify the threading of the tubing through the IV pump. • Determine if the clamp on the IV tubing is released A client with arthritis has been receiving treatment with naproxen and now reports ongoing stomach pain, increasing weakness, and fatigue. Which laboratory test should the nurse monitor? • Sed rate (ESR) • Hemoglobin • Calcium • Osmolality. After receiving the Braden scale findings of residents at a long-term facility, the charge nurse should to tell the unlicensed assistive personnel (UAP) to prioritize the skin care for which client? • An older adult who is unable to communicate elimination needs. • An older man whose sheets are damped each time he is turned. • A woman with osteoporosis who is unable to bear weight. • A poorly nourished client who requires liquid supplement. A client with acute renal failure (ARF) is admitted for uncontrolled type 1 diabetes Mellitus and hyperkalemia. The nurse administers an IV dose of regular insulin per sliding scale. Which intervention is the most important for the nurse to include in this client’s plan of care? • Monitor the client’s cardiac activity via telemetry. • Maintain venous access with an infusion of normal saline. • Assess glucose via fingerstick q4 to 6 hours. • Evaluate hourly urine output for return of normal renal function. A client with C-6 spinal cord injury rehabilitation. In the middle of the night the client reports a severe, pounding headache, and has observable piloerection or “goosebumps”. The nurse should asses for which trigger? • Loud hallway noise. • Fever • Full bladder • Frequent cough. A nurse working on an endocrine unit should see which client first? • An adolescent male with diabetes who is arguing about his insulin dose. • An older client with Addison’s disease whose current blood sugar level is 62mg/dl (3.44 mmol/l). • An adult with a blood sugar of 384mg/dl (21.31mmol/l) and urine output of 350 ml in the last hour. • A client taking corticosteroids who has become disoriented in the last two hours. Four hours after surgery, a client reports nausea and begins to vomit. The nurse notes that the client has a scopolamine transdermal patch applied behind the ear. What action should the nurse take? • Reposition the transdermal patch to the client’s trunk. • Remove the transdermal patch until the vomiting subsides. • Notify the healthcare provider of the vomiting. • Explain that this is a side effect of the medication in the patch. The nurse identifies an electrolyte imbalance, an elevated pulse rate, and elevated BP for a client with chronic kidney disease. Which is the most important action for the nurse to take? • Monitor daily sodium intake. • Record usual eating patterns. • Measure ankle circumference. • Auscultate for irregular heart rate. A client with persistent low back pain has received a prescription for electronic stimulator (TENS) unit. After the nurse applies the electrodes and turns on the power, the client reports feeling a tingling sensation. How should the nurse respond? • Determine if the sensation feels uncomfortable. • Decrease the strength of the electrical signals. • Remove electrodes and observe for skin redness. • Check the amount of gel coating on the electrodes. A female client is extremely anxious after being informed that her mammogram was abnormal and needs to be repeated. Client is tearful and tells the nurse her mother died of breast cancer. What action should the nurse take? • Provide the client with information about treatment options for breast cancer. • Reassure the client that the final diagnosis has not been made. • Encourage the client to continue expressing her fears and concerns. • Suggest to the client that she seek a second opinion. The psychiatric nurse is talking to a newly admitted client when a male client diagnosed with antisocial behavior intrudes on the conversation and tells the nurse, “I have to talk to you right now! It is very important!” how should the nurse respond to this client? • Put his behavior on extinction and continue talking with the newly admitted. • Inform him that the nurse is busy admitting a new client and will talk to him later. • Encourage him to go to the nurse’s station and talk with another nurse. • Introduce him to the newly admitted client and ask him to him to join in the conversation. The charge nurse is planning for the shift and has a registered nurse (RN) and a practical nurse (PN) on the team. Which client should the charge nurse assign to the RN? • A 64 year old client who had a total hip replacement the previous day. • A 75 year old client with renal calculi who requires urine straining. • An adolescent with multiple contusions due to a fall that occurred 2 days ago. • A 30 year old depressed client who admits to suicide ideation. A female client presents in the Emergency Department and tells the nurse that she was raped last night. Which question is most important for the nurse to ask? • Does she knows the person who raped her? • Has she taken a bath since the raped occurred? • Is the place where she lived a safe place? • Did she report the rape to the police Department? While caring for a client’s postoperative dressing, the nurse observes purulent drainage at the wound. Before reporting this finding to the healthcare provider, the nurse should review which of the client’s laboratory values? • Serum albumin • Creatinine level • Culture for sensitive organisms. • Serum blood glucose (BG) level The nurse is demonstrating correct transfer procedures to the unlicensed assisted personnel (UAP) working on a rehabilitation unit. The UAPs ask the nurse how to safely move a physically disabled client from the wheelchair to a bed. What action should the nurse recommended? • Hold the client at arm’s length while transferring to better distribute the body weight. • Apply the gait belt around the client’s waits once standing position has been assumed. • Place a client’s locked wheelchair on the client’s strong side next to the bed. • Pull the client into position by reaching from the opposite side of the bed. A client who is experiencing musculoskeletal pain receives a prescription for ketorolac 15mg IM q6 hours. The medication is depended in a 30 mg/ml pre-filled syringe. Which action should the nurse implement when giving the medication? • Administer the entire pre-filled syringe deep in the dorsogluteal site. • Use a separate syringe to remove 15mg from the pre-filled syringe and give in the back of the arm. • Waste 0.5 ml from the pre-filled syringe and inject the medication in the ventrogluteal site. • Call the healthcare provider to request a prescription change to match the dispensed 30mg dose. A client with a lower respiratory tract infection receives a prescription for ciprofloxacin 500mg PO q 12hours. When the client request an afternoon snack, which dietary choice should the nurse provide? • Vanilla-flavored yogurt • Low fat chocolate milk. • Calcium fortified juice • Cinnamon applesauce The healthcare provider prescribes a low-fiber diet for a client with ulcerative colitis. Which food selection would indicate to the nurse the client understands they prescribed diet? • Roasted turkey canned vegetables • Baked potatoes with skin raw carrots • Pancakes whole-grain cereal's • Roast pork fresh strawberries An adult client with schizophrenia begin treatment three days ago with the Antipsychotic risperidone. The client also received prescription for trazodone as needed for sleep and clonazepam as needed for severe anxiety. When the client reports difficulty with swallowing, what action should the nurse take? • Obtain a prescription for an anticholinergic medication • Determine how many hours declined slept last night • Administer the PRN prescription for severe anxiety • Watch the thyroid cartilage move while the client swallows One year after being discharged from the burn trauma unit, a client with a history of 40% full-thickness burns is admitted with bone pain and muscle weakness. Which intervention should the nurse include in the clients plan of care? • Encourage Progressive active range of motion • Teach need for dietary and supplementary vitamin D3 • Explain the need for skin exposure to sunlight without sunscreen • Instruct the client to use of muscle strengthening exercises When teaching a group of school-age children how to reduce the risk of Lyme disease which instruction should the camp nurse include? • Wash hands frequently • Avoid drinking lake water • Wear long sleeves and pants • Do not share personal products A native-American male client diagnosed with pneumonia, states that in addition to his prescribed medical treatment of IV antibiotics he wishes to have a spiritual cleaning performed. Which outcome statement indicates that the best plan of care was followed? • Identifies his ethnocentric values and behaviors • States an understanding of the medical treatment • Participated actively in all treatments regimens • Expresses a desire for cultural assimilation A male client with cancer is admired to the oncology unit and tells the nurse that he is in the hospital for palliative care measures. The nurse notes that the client’s admission prescription include radiation therapy. What action should the nurse implement? • Ask the client about his expected goals for the hospitalization • Explain the palliative care measures can be provided at home • Notify do radiation department to withhold the treatment for now • Determine if the client wishes to cancel further radiation treatment A client with myasthenia Gravis (MG) is receiving immunosuppressive therapy. Review recent laboratory test results show that the client’s serum magnesium level has decreased below the normal range. In addition to contacting the healthcare provider, what nursing action is most important? • Check the visual difficulties • Note most recent hemoglobin level • Assessed for he and Hand joint pain • Observe rhythm on telemetry monitor A young adult female presents at the emergency center with acute lower abdominal pain. Which assessment finding is most important for the nurse to report to the healthcare provider? • Pain scale rating at 9 on a 0-10 scale • Last menstrual period was 7 weeks ago • Reports white curdy vaginal discharge • History of irritable bowel syndrome IBS The nurse is assessing a postpartum client who is 36 hours post-delivery. Which finding should the nurse report to the healthcare provider? • White blood count of 19,000 mm3 • Oral temperature of 100.6 F • Fundus deviated to the right side • Breasts are firm when palpated A nurse who is working in the emergency department triage area is presented with four clients at the same time. The client presented with which symptoms requires the most immediate intervention by the nurse? • Low-grade fever, headache, and malaise for the past 72 hours • Unable to bear weight on the left foot, with the swelling and bruising • Chest discomfort one hour after consuming a large, spicy meal • One-inch bleeding laceration on the chain of the crying five-year-old The nurse is planning to assess a client's oxygen saturation to determine if additional oxygen is needed via nasal cannula. The client has a bilateral below-the-knee amputation and pedal pulses that are weak and threaty. What action should the nurse take? • Document that an accurate oxygen saturation reading cannot be obtained • Elevate to client's hands for five minutes prior to obtaining a reading from the finger • Increase the oxygen based on the clients breathing patterns and lung sounds • Place the oximeter clip on the ear lobe to obtain the oxygen saturation reading A young adult male who is being seen at the employee health care clinic for an annual assessment tell the nurse that his mother was diagnosed with schizophrenia when she was his age and that life with a schizophrenic mother was difficulty indeed. Which response is best for the nurse to provide? • Ask the client if he is worried about becoming schizophrenic at the age his mother was diagnosed. • Encourage the client to seek genetic counseling to determine his risk for mental illness. • Informed the client that his mother schizophrenic has affected his psychological development. • Tell the client that mental illness has a familial predisposition so he should see a psychiatrist. A client on a long-term mental health unit repeatedly takes own pulse regardless of the circumstance. What action should the nurse implement? • Overlook the client’s behavior. • Distract client to interfere with the ritual. • Ask why the client checks the pulse. • Hold client’s hand to stop the behavior. A client is discharged with automated peritoneal dialysis (PD) to be used nightly…which instructions should the nurse include? • Wash hands before cleaning exit site • Keep the head of the bed flat at night • Feel for a thrill and a distal pulse nightly • Do not get up if fluid is left in the abdomen The charge nurse observes the practical nurse (PN) apply sterile gloves in preparation for performing a sterile dressing change. Which action by the PN requires correction by the charge nurse? • Opening the package • Picking up the second glove • Picking up the first glove • Positioning of the table A male client reports to the clinic nurse that he has been feeling well and is often “dizzy” his blood pressure is elevated. Based on this findings, this client is at a greatest risk for which pathophysiological condition? • Stroke • Renal failure • Left ventricular hypertrophy • Pulmonary hypertension The nurse ask the parent to stay during the examination of a male toddler’s genital area. Which intervention should the nurse implement? • Examine the genitalia as the last part of the total exam. • Use soothing statements to facilitate cooperation • Allow the child to keep underpants on to examine genitalia • Work slowly and methodically so not to stress the child The nurse is changing a client’s IV tubing and closes the roller clamp on the new tubing setup when the bag of solution is….which action should the nurse take to ensure adequate filling of the drip chamber? • Lower the IV bag to a flat surface • Compress the drip chamber • Open the roller clamp • Squeeze the bag of IV solution An Insulin infusion for a client with diabetes mellitus who is experiencing hyperglycemic hyperosmolar…in addition to the client’s glucose, which laboratory value is most important for the nurse to monitor? • Serum potassium • Urine ketones • Urine albumin • Serum protein A young adult who is hit with a baseball bat on the temporal area of the left skull is conscious when admitted to the ED and is transferred to the Neurological Unit to be monitored for signs of closed head injury. Which assessment finding is indicative of a developing epidural hematoma? • Altered consciousness within the first 24 hours after injury. • Cushing reflex and cerebral edema after 24 hours • Fever, nuchal rigidity and opisthotonos within hours • Headache and pupillary changes 48 hours after a head injury In planning strategies to reduce a client's risk for complications following orthopedic surgery, the nurse recognizes which pathology as the underlying cause of osteomyelitis? • infectious process • metastatic process • autoimmune disorder • inflammatory disorder A client with a serum sodium level of 125 meq/mL should benefit most from the administration of which intravenous solution? • 0.9% sodium chloride solution (normal saline) • 0.45% sodium chloride solution (half normal saline) • 10% Dextrose in 0.45% sodium chloride • 5% dextrose in 0.2% sodium chloride A client with Alzheimer’s disease falls in the bathroom. The nurse notifies the charge nurse and completes a fall follow-up assessment. What assessment finding warrants immediate intervention by the nurse? • Urinary incontinence • Left forearm hematoma • Disorientation to surroundings • Dislodge intravenous site The nurse is triaging clients in an urgent care clinic. The client with which symptoms should be referred to the health care provider immediately? • headache, photophobia, and nuchal rigidity • high fever, skin rash, and a productive cough • nausea, vomiting, and poor skin turgor • malaise, fever, and stiff, swollen joints An adult male is brought to the emergency department by ambulance following a motorcycle accident. He was not wearing a helmet and presents with periorbital bruising and bloody drainage from both ears. Which assessment finding warrants immediate intervention by the nurse? • Rebound abdominal tenderness • nausea and projectile vomit • rib pain with deep inspiration • diminished bilateral breath sounds After placement of a left subclavian central venous catheter (CVC), the nurse receives report of the x-ray findings that indicate the CVC tip is in the client’s superior vena cava. Which action should the nurse implement? • Initiate intravenous fluid as prescribed • Notify the HCP of the need to reposition the catheter • Remove the catheter and apply direct pressure for 5 minute • Secure the catheter using aseptic technique The nurse has received funding to design a health promotion project for African-American women who are at risk for developing breast cancer. Which resource is most important in designing this program? • A listing of African-American women so live in the community • Participation of community leaders in planning the program • Morbidity data for breast cancer in women of all races • Technical assistance to produce a video on breast self-examination. The home care nurse provide self-care instruction for a client chronic venous insufficiency cause by deep vein thrombosis. Which instructions should the nurse include in the client’s discharge teaching plan? Select all that apply • Avoid prolonged standing or sitting • Use recliner for long period of sitting • continue wearing elastic stocking • Maintain the bed flat while sleeping • Cross legs at knee but not at ankle The nurse is interviewing a client with schizophrenia. Which client behavior requires immediate intervention? • Lip smacking and frequent eye blinking • Shuffling gait and stooped posture • Rocks back and forth in the chair • Muscle spasms of the back and neck A male client was transferred yesterday from the emergency department to the telemetry unit because he had ST depression and resolved chest pain. When his EKG monitor alarms for ventricular tachycardia (VT), what action should the nurse take first? • Determine the client’s responsiveness and respirations • Bring the crash cart to the room to defibrillate the client. • Immediately initiate chest compressions. • Notify the emergency response team A client with a large pleural effusion undergoes a thoracentesis. Following the procedure, which assessment finding warrants immediate intervention by the nurse? • The client has asymmetrical chest wall expansion • The clients complain of pain at the insertion site • The client chest’s x-ray indicates decreased pleural effusion • The client’s arterial blood gases are pH 7.35, PaO2 85, Pa CO2 35, HCO3 26 The nurse is preparing a client for discharge from the hospital following a liver transplant. Which instruction is most important for the nurse to include in this client’s discharge teaching plan? • Monitor for an elevated temperature • Measure the abdominal girth daily • Report the onset of sclera jaundice • Keep a record of daily urinary output The nurse is conducting health assessments. Which assessment finding increases a 56-year-old woman’s risk for developing osteoporosis? • Body mass index of (BMI) of 31 • 20 pack-year history of cigarette smoking • Birth control pill usage until age 45 • Diabetes mellitus in family history A young couple who has been unsuccessful in conceiving a child for over a year is seen in the family planning clinic. During an initial visit, which intervention is most important for the nurse to implement? • Determine current sexual practice • Prepare a female client for an ultrasound • Request an sperm sample for ovulation • Evaluate hormone levels on both client The nurse administers an oral antiviral to a client with shingles. Which finding is most important for the nurse report to the health care provider? • Decreased white blood cell count • Pruritus and muscle aches • Elevated liver function tests • Vomiting and diarrhea A client in the intensive care unit is being mechanically ventilated, has an indwelling urinary catheter in place, and is exhibiting signs of restlessness. Which action should the nurse take first? • Review the heart rhythm on cardiac monitors • Check urinary catheter for obstruction • Auscultated bilateral breath sounds • Give PRN dose of lorazepam (Ativan) The nurse makes a supervisory home visit to observe an unlicensed assistive personnel (UAP) who is providing personal care for a client with Alzheimer’s disease. The nurse observes that whenever the client gets upset, the UAP changes the subject. What action should the nurse take in response to this observation? • Tell the UAP to offer more choices during the personal care to prevent anxiety • Meet with the UAP later to role model more assertive communication techniques • Assume care of the client to ensure that effective communication is maintained. • Affirm that the UAP is using and effective strategy to reduce the client’s anxiety. An older female who ambulate with a quad-cane prefer to use a wheel chair because she has a halting and unsteady gait at times. Which interventions should the nurse implement? (Select all that apply) • Move personal items within client’s reach • Lower bed to the lower possible position • Give directions to call for assistance • Assist client to the bathroom in 2 hours. • Encourage the use of the wheelchair • Raise all bed rails when the client is resting In evaluating the effectiveness of a postoperative client’s intermittent pneumatic compression devices, which assessment is most important for the nurse to complete? • Evaluate the client’s ability to use an incentive spirometer • Monitor the amount of drainage from the client’s incision • Observe both lower extremities for redness and swelling • Palpate all peripheral pulse points for volume and strength A nurse stops at the site of a motorcycle accident and finds a young adult male lying face down in the road in a puddle of water. It is raining, no one is available to send for help, and the cell phone is in the car about 50 feet away. What action should the nurse take first? • Examine the victim’s body surfaces for arterial bleeding • Stabilize the victim’s neck and roll over to evaluate his status • Return to the car to call emergency response 911 for help • Open the airway and initiate resuscitative measures During a well-baby, 6-month visit, a mother tells the nurse that her infant has had fewer ear infections than her 10-year-old daughter. The nurse should explain that which vaccine is likely to have made the difference in the siblings’ incidence of otitis media? • Varicella Virus Vaccine Live • Hemophilic Influenza Type B (HiB) vaccine • Pneumococcal vaccine • Palivizumab vaccine for RSV Which medication should the nurse anticipate administering to a client who is diagnosed with myxedema coma? • Intravenous administration of thyroid hormones • Oral administration of hypnotic agents • Intravenous bolus of hydrocortisone • Subcutaneous administration of vitamin k The nurse who works in labor and delivery is reassigned to the cardiac care unit for the day because of a low census in labor and delivery. Which assignments is best for the nurse to give this nurse? • Transfer a client to another unit • Monitor the central telemetry • Perform the admission • Assist cardiac nurses with their assignments A client who had an emergency appendectomy is being mechanically ventilated, and soft wrist restrain are in place to prevent self extubation. Which outcome is most important for the nurse to include in the client’s plan of care? • Understand pain management scale • Maintain effective breathing patterns • Absence of ventilator associated pneumonia • No injuries refer to soft restrains occur After a routine physical examination, the healthcare admits a woman with a history of Systemic Lupus Erythematous (SLE) to the hospital because she has 3+ pitting ankle edema and blood in her urine. Which assessment finding warrants immediate intervention by the nurse? • Blood pressure 170/98 • Joint and muscle aches • Urine output 300 ml/hr • Dark, rust-colored urine The nurse is explaining the need to reduce salt intake to a client with primary hypertension. What explanation should the nurse provide? • High salt can damage the lining of the blood vessels • Too much salt can cause the kidneys to retain fluid • Excessive salt can cause blood vessels to constrict • Salt can cause information inside the blood vessels In assessing a pressure ulcer on a client’s hip, which action should the nurse include? • Determine the degree of elasticity surrounding the lesion • Photograph the lesion with a ruler placed next to the lesion • Stage the depth of the ulcer using the Braden numeric scale • Use a gloved finger to palpate for tunneling around the lesion A nurse is planning discharge care for a male client with metastatic cancer. The client tells the nurse that he plans to return to work despite pain, fatigue, and impending death. Which goals is most important to include in this client’s plan of care? • Implements decisions about future hospices services within the next 3 months. • Marinating pain level below 4 when implementing outpatient pain clinic strategies. • Request home health care if independence become compromised for 5 days. • Arranges for short term counseling stressors impact work schedule for 2 weeks. A client who had an open cholecystectomy two weeks ago comes to the emergency department with complaints of nausea, abdominal distention, and pain. Which assessment should the nurse implement? • Auscultate all quadrant of the abdomen. • Perform a digital rectal exam • Palpate the liver and spleen • Obtain a hemoccult of the client’s stool The nurse is caring for several clients on a telemetry unit. Which client should the nurse assess first? The client who is demonstrating • A paced rhythm with 100% capture after pacemaker replacement • Normal sinus rhythm and complaining of chest pain • Atrial fibrillation with congestive heart failure and complaining of fatigue • Sinus tachycardia 3 days after a myocardial infarction The nurse is evaluating the health teaching of a female client with condyloma acuminate. Which statement by the client indicates that teaching has been effective? • Early treatment is very effective • I will clean my hot tub better • These warts are caused by a fungus • I need to have regular pap smears While the nurse is conducting a daily assessment of an older woman who resides in a long-term facility, the client begins to cry and tells the nurse that her family has stopped calling and visiting. What action should the nurse take first? • Ask the client when a family member last visited her. • Determine the client’s orientation to time and space • Review the client’s record regarding social interactions • Reassure the client of her family’s love for her A female client with severe renal impairment is receiving enoxaparin (lovenox) 30 mg SUBQ BID. Which laboratory value due to enoxaparin should the nurse report to the healthcare provider? • creatinine clearance 25 mL/ minute • calcium 9 mg/dl • hemoglobin 12 grams/dl • partial thromboplastin time (PTT) 30 seconds The nurse is assigned to care for clients on a medical unit. Based on the notes taken during the shift report, which client situation warrants the nurse’s immediate attention? • A 10-year-old who is receiving chemotherapy and the infusion pump is beeping • A young adult with Crohn’s disease who reports having diarrheal stools • An older adult with type 2 diabetes whose breakfast tray arrives 20 minutes late • A teenager who reports continued pain 30 minutes after receiving an oral analgesic A nurse is conducting a physical assessment of a young adult. Which information provides the best indication of the individual nutritional status? • Condition of hair, nails, and skin • A 24-hour diet history • History of a recent weight loss • Status of current petite The nurse is preparing to administer an infusion of amino acid-dextrose total parenteral nutrition (TPN) through a central venous catheter (CVC) line. Which action should the nurse implement first? • Check the TPN solution for cloudiness • Attach the IV tubing to the central line • Set the infusion pump at the prescribed rate • Prime the IV tubing with TPN solution A newly admitted client vomits into an emesis basin as seen in the picture. The nurse should consult with the healthcare provider before administering which of the client’s prescribes medications? • Clopidogrel (Plavix), an antiplatelet agent, given orally • Nitroglycerin (nitro-dur), an antianginal, to be given transdermally • Methylprednisolone (solu-medrol), a corticosteroid, to be given IV • Furosemide (lasix), a loop diuretic, to be given intravenously • Enoxaparin (lovenox), a low-molecular weight heparin to be given subcutaneous A client diagnosed with bipolar disorder is going home on a week-end pass. Which suggestions should give the client’s family to help them prepare for the visit? • Encourage the family to plan daily activities to keep the client busy • Have friends and family visit the client at a welcome home party • Discuss the importance of continuing the usual at-home activities • Instruct family to monitor the client’s choice of television programs On a busy day, one hour after the shift report is completed, the charge nurse learns that a female staff nurse who lives one hour away from the hospital forgot her prescription eye glasses at home. What action should the charge nurse take? • Encourage the nurse to purchase reading glasses in the hospital gift shop • Request another nurse to assist the staff nurse with her documentation • Ask the nurse to return home and get her prescription eyeglasses for work. • Tell the staff nurse to take a day off and change her weekly work schedule When five family members arrive at the hospital, they all begin asking the nurse questions regarding the prognosis of their critically ill mother. What intervention should the nurse implement first? • Include the family in client’s care • Request the chaplain’s presence • Ask the family to identify a specific spokesperson • Page the healthcare provider to speak with family. The nurse reviews the signs of hypoglycemia with the parents of a child with Type I diabetes mellitus. The parents correctly understand signs of hypoglycemia if they include which symptoms? • Fruity breath odor • Polyphagia • Diaphoresis • Polydipsia The nurse is caring for a client with hypovolemic shock who is receiving two units of packed red blood cells (RBCs) through a large bore peripheral IV. What action promotes maintenance of the client’s cardiopulmonary stability during the blood transfusion? • Increase the oxygen flow via nasal cannula if dyspnea is present. • Place in a Trendelenburg position to increase cerebral blood flow • Monitor capillary glucose measurements hourly during transfusion. • Encourage increased intake of oral fluid to improve skin turgor. Which information is more important for the nurse to obtain when determining a client’s risk for (OSAS)? • Body mass index • Level of consciousness • Self-description of pain • Breath sounds During the transfer of a client who had major abdominal surgery this morning, the post anesthesia care unit (PACU) nurse reports that the client, who is awake and responsive continues to report pain and nausea after receiving morphine 2 mg IV and ondansetron 4 mg IV 45 mints ago. Which elements of SBAR communication are missing from the report given by the PACU nurse? (Select all that apply) • Situation • Background • Assessment • Recommendation • Rationales. The nurse is triaging victims of a tornado at an emergency shelter. An adult woman who has been wandering and crying comes to the nurse. What action should the nurse take? • Check the client’s temperature, blood sugar, and urine output. • Transport the client for laboratory client for laboratory test and electrocardiogram (EKG) • Delegate care of the crying client to an unlicensed assistant • Send the client to the shelter’s nutrient center to obtain water and food. A client in septic shock has a double lumen central venous catheter with one liter of 0.9% Normal Saline Solution infusing at 1 ml/hour through one lumen and TPN infusing at 50 ml/hr. through one port. The nurse prepared newly prescribed IV antibiotic that should take 45 mints to infuse. What intervention should the nurse implement? • Use a secondary port of the Normal Saline solution to administer the antibiotic. • Add the antibiotic to the TPN solution, and continue the normal saline solution. • Stop the TPN infusion for the time needed to administer the prescribed antibiotic. • Add the antibiotic to the Normal Saline solution and continue both infusions. A male client returns to the mental health clinic for assistance with his anxiety reaction that is manifested by a rapid heartbeat, sweating, shaking, and nausea while driving over the bay bridge. What action I the treatment plan should the nurse implement? • Tell the client to drive over the bridge until fear is manageable • Teach client to listen to music or audio books while driving • Encourage client to have spouse drive in stressful places. • Recommend that the client avoid driving over the bridge. Which intervention should the nurse include in the plan of care for a client with leukocytosis? • Avoid intramuscular injections • Monitor temperature regularly • Assess skin for petechiae or bruising • Implement protective isolation measures The nurse is teaching a client about the antiulcer medications ranitidine which was… statement best describes the action of this drug? • It blocks the effects of histamine, causing decreased secretion of acid • Ranitidine will neutralize gastric acid and decrease gastric pH • This drug provides a protective coating over the gastric mucosa • It effectively blocks 97% of the gastric acid secreted in the stomach A client with superficial burns to the face, neck, and hands resulting from a house fire…which assessment finding indicates to the nurse that the client should be monitored for carbon monoxide…? • Expiratory stridor and nasal flaring • Mucous membranes cherry red color • Carbonaceous particles in sputum • Pulse oximetry reading of 80 percent A female client who was mechanically ventilated for 7 days is extubated. Two hours later…productive cough, and her respirations are rapids and shallow. Which intervention is most important? • Review record of recent analgesia • Provide frequent pulmonary toilet • Prepare the client for intubation • Obtain STAT arterial blood gases The nurse is assessing the emotional status of a client with Parkinson’s disease. Which client finding is most helpful in planning goals to meet the client’s emotional needs? • Cries frequently during the interview • Stares straight ahead without blinking • Face does not convey any emotion • Uses a monotone when speaking When changing a diaper on a 2-day-old infant, the nurse observes that the baby’s legs are… this finding, what action should the nurse take next? • Notify the healthcare provider • Continue care since this is a normal finding • Document the finding in the record • Perform range of motion to the joint The nurse is presenting information about fetal development to a group of parents with…when discussing cephalocaudal fetal development, which information should the nurse gives the parents? • A set order in fetal development is expected • Growth normally occurs within one organ at a time • Development progress from head to rump • Organ formation is directed by brain development A client has a prescription for lorazepam 2mg for alcohol withdrawal symptoms. Which finding… the client? • Blood pressure 149/101 • Irregular pulse rate of 80 • Oral temperature is 98.9 F (37.1 C) • Pain rated 7 on scale 1-10 A male client is discharged from the intensive care unit following a myocardial infarction, and the healthcare provider low-sodium diet. Which lunch selection indicates to the nurse that this client understands the dietary restrictions? • Turkey salad sandwich. • Clam chowder • Macaroni and cheese • Bacon, lettuce, and tomato sandwich A client is admitted with a wound on the right hand and associated cellulitis. In assessing the client’s hand, which finding required most immediate follow-up by the nurse? • Cyanotic nailbeds • Localized tenderness • Diffuse erythema • Skin hot to touch When conducting diet teaching for a client who was diagnosed with hypoparathyroidism, which foods should the nurse encourage the client to eat? • Yogurt. • Processed cheese. • Nuts • Fresh turkey • Fresh chicken The nurse is assessing a middle-aged adult who is diagnosed with osteoarthritis. Which factor in this client’s history is a contributor to the osteoarthritis? • Long distance runner since high school. • Lactose intolerant since childhood • Photosensitive to a drug currently taking • Recently treated for deep vein thrombosis When assessing a male client, the nurse notes that he has unequal lung expansion. What conclusion regarding this finding is most likely to be accurate? The client has • A collapsed lung • A history of COPD • A chronic lung infection • Normally functioning lungs The nurse requests a meals tray for a client follows Mormon beliefs and who is on clear liquid diet following abdominal surgery. Which meal item should the nurse request for this client? (Select all that apply) • Apple juice • Chicken broth. • Hot chocolate • Orange juice • Black coffee The healthcare provider prescribes oxycodone/ aspirin 1 tab PO every 4h as needed for pain, for a client with polycystic kidney disease. Before administering this medication, which component of the prescription should the nurse question? • Aspirin content. • Dose • Route • Risk for addiction Which interventions should the nurse include in a long-term plan of care for a client with COPD? • Reduce risk factors for infection • Limit fluid intake to reduce secretions • Use diaphragmatic breathing to achieve better exhalation • Administer high flow oxygen during sleep A client delivers a viable infant, but begins to have excessive uncontrolled vaginal…notifying the health care provider of the clients’ condition, what information is most…. • Maternal blood pressure • Maternal apical pulse rate • Time Pitocin infusion completed • Total amount of Pitocin infused An infant born to a heroin-addicted mother is admitted to the neonatal care unit. What behaviors can…to exhibit? • Irritability and a high-pitched cry • Lethargy and poor suck • Facial abnormalities and microcephaly • Low birth weight and intrauterine growth retardation A multigravida, full-term, laboring client complains of “back labor”. Vaginal examination reveals that the client’s 3 cm with 50% effacement and the fetal head is at -1 station. What should the nurse implement? • Turn the client to a lateral position • Apply counter-pressure to the sacral area • Notify the scrub nurse to prepare the OR • Ambulate the client between contractions A postpartal client complains that she has the urge to urinate every hour but is only able to void a small amount. What interventions provides the nurse with the most useful information? • Catheterize for residual urine after next voiding • Initiate a perineal pad count • Assess for a perineal hematoma • Determine the client’s usual voiding pattern A client is receiving oxytocin (Pitocin) to augment early labor. Which assessment is most important time the infusion rate is increases? • Contraction pattern • Blood pressure • Infusion site • Pain level To obtain an estimate of a client’s systolic B/P. What action should the nurse take first? • Palpate the client’s brachial pulse • Pump up the blood pressure cuff • Position the stethoscope diaphragm • Release the blood pressure cuff valve A client is admitted to isolation with the diagnosis of active tuberculosis (TB). Which infection control measures should the nurse implement? • Negative pressure environment • Contact precautions • Droplet precautions • Protective environment A male client is returned to the surgical unit following a left nephrectomy and is medicated with morphine. His dressing has a small amount of bloody drainage, and a Jackson-Pratt bulb surgical drainage device is in place. Which interventions is most important for the nurse to include in this clients plan of care? • Monitor urine output hourly. • Assess for back muscle aches • Record drainage from drain • Obtain body weight daily A primigravida client is 36 weeks gestation is admitted to labor and delivery unit because her membranes ruptured 30minutes ago. Initial assessment indicates 2cm dilation, 50% effaced, -2 station, vertex presentation greenish colored amniotic fluid, and contractions occurring 3-5 minutes with a low FHR after the last contraction peaks: • Administer Oxygen via face mask • Apply an internal fetal heart monitor • Notify the healthcare provider • Use a vibroacoustic stimulator Following a motor vehicle collision (MCV), a male adult in severe pain is brought to the emergency department via ambulance. His injured left leg is edematous, ecchymotic around the impact of injury on the thigh, and shorter than his right leg. Based on these findings, the client is at greatest risk for which complication? • Arterial ischemia • Tissue necrosis • Fat embolism • Nerve damage A 2-year-old is bleeding from a laceration on the right lower extremity that occurred as the result of a motor vehicle collision. The nurse is selecting supplies to start an IV access. Which assessment finding is most significant in the nurse's selection of catheter size? • Thready brachial pulse. • Respirations of 24/minute • Right foot cool to touch • Swelling at the site of injury A client with a recent colostomy expresses concern about the ability to control flatus. Which intervention is most important for the nurse to include in the client’s plan of care? • Adhere to a bland diet whenever planning to eat out • Decrease fluid intake at meal times • Avoid foods that caused gas before the colostomy • Eliminate foods high in cellulose A male client arrives at the clinic with a severe sunburn and explains that he did not use sun screen because it was an overcast day. Large blisters are noted over his back and chest and his shirt is soaked with serosanguinous fluid. Which assessment finding warrants immediate intervention by the nurse? • Hypotension. • Fever and chills • Dizziness • Headache A client with polycystic kidney disease (PKD) receiving antibiotics for an infected cyst is experiencing severe pain. What action should the nurse implement? • Hold the next dose of antibiotic until contacting the healthcare provider • Teach the client how to use a dry heating pad over the painful area • Encourage the client to practice pelvic floor exercises every hour • Assist the client to splint the site by applying an abdominal binder Which statement is accurate regarding the pathological changes in the pulmonary system associated with acute (adult) respiratory distress syndrome (ARDS)? • Capillary hydrostatic pressure exceeds colloid osmotic pressure, producing interstitial edema • A high ventilation-to-perfusion ratio is characteristic of affected lung fields in ARDS • Functional residual capacity and lung compliance increase as the disease progresses • Interstitial edema that occurs due to capillary fluid shifts is usually more serious than alveolar edema An adult male who was admitted two days ago following a cerebrovascular accident (CVA) is confused and experiencing left-side weakness. He has tried to get out of bed several times, but is unable to ambulate without assistance. Which intervention is most important for the nurse to implement? • Ask a family member to sit with the client • Apply bilateral soft wrist restraints • Assign staff to check client q15 minutes • Install a bed exit safety monitoring device A client in her first trimester of pregnancy complains of nausea. Which complementary therapy should the nurse recommend? • Drink chamomile tea at breakfast and in the evening. • Eat food high in garlic with the evening meal • Join a yoga class that meets at least weekly • Increase cocoa in the diet and drink before bedtime // When gathering for a group therapy session at 1400 hours, a female client complains to the nurse that a smoking break has not been allowed all day. The nurse responds that 15 minute breaks were called over the unit intercom after breakfast and after lunch. The nurse is using what communication technique in responding to the client? • Doubt • Observation • Confrontation • Reflection A female client with rheumatoid arthritis (RA) comes to the clinic complaining of joint pain and swelling. The client has been taking prednisone (Deltasone) and ibuprofen (Motrin Extra Strength) every day. To assist the client with self-management of her pain, which information should the nurse obtain? • Presence of bruising, weakness, or fatigue • Therapeutic exercise included in daily routine. • Average amount of protein eaten daily • Existence of gastrointestinal discomfort The charge nurse of the Intensive Care Unit is making assignments for the permanent staff and one RN who was floated from a medical unit. The client with which condition is the best to assign to the float nurse? • Diabetic ketoacidosis and titrated IV insulin infusion • Emphysema extubated 3 hours ago receiving heated mist • Subdural hematoma with an intracranial monitoring device • Acute coronary syndrome treated with vasopressors A client admitted to the emergency center had inspiratory and expiratory wheezing, nasal flaring, and thick, tenacious sputum secretions observed during the physical examination. Based on these assessment findings, what classification of pharmacologic agents should the nurse anticipate administering? • Beta blockers • Bronchodilators • Corticosteroids • Beta-adrenergics The home health nurse is assessing a male client who has started peritoneal dialysis (PD) 5 days ago. Which assessment finding warrants immediate intervention by the nurse? • Finger stick blood glucose 120 mg/dL post exchange • Arteriovenous (AV) graft surgical site pulsations. • Anorexia and poor intake of adequate dietary protein • Cloudy dialysate output and rebound abdominal pain A client’s telemetry monitor indicates ventricular fibrillation (VF). What should the nurse do first? • Administer epinephrine IV • Give an IV bolus of amiodarone • Provide immediate defibrillation • Prepare for synchronized cardioversion In conducting a health assessment, the nurse determines that both parents of a child with asthma smoke cigarettes. What recommendation is best to the nurse to recommend to the parents? • avoid smoking in the house • stop smoking immediately • decrease the number of cigarettes smoke daily • obtain nicotine patches to assist in smoking sensation A client who is schedule for an elective inguinal hernia repair today in day surgery is seem eating in the waiting area. What action should be taken by the nurse who is preparing to administer the preoperative medications? • Review the surgical consent with the client • Explain that vomiting can occur during surgery • Remove the food from the client • Withhold the preoperative medication The nurse is developing a plan of care for a middle-aged woman who is diagnosed with type 2 diabetes mellitus (DM). To lower her blood glucose and increase her serum high-density lipoprotein (HDL) levels, which instruction is most important for the nurse to provide? • Exercise at least three times weekly • Monitor blood glucose levels daily • Limit intake of foods high in saturated fat • Learn to read all food product labels A client who has been in active labor for 12 hours suddenly tells the nurse that she has a strong urge to have a bowel movement. What action should the nurse take? • Allow the client to use a bedpan. • Assist the client to the bathroom • Perform a sterile vaginal exam • Explain the fetal head is descending. The nurse assesses a 78-year-old male client who has left sides heart failure. Which symptoms would the nurse expect this client to exhibit? • Dyspnea, cough, and fatigue. • Hepatomegaly and distended neck veins • Pain over the pericardium and friction rub. • Narrowing pulse pressure and distant heart sounds. A female client comes to the clinic complaining of fatigue and inability to sleep because she is the full-time caretaker for 22-year-old son who was paralyzed by a motor vehicle collision. She adds that her husband left her because he says he can’t take her behavior any more since all she does is care for their son. What intervention should the nurse implement? • Schedule a home visit in the afternoon to assess the son and client role as caregiver. • Acknowledge the client’s stress and suggest that she consider respite care. • Provide feedback to the client about her atonement for guilt about her son’s impairment. • Teach the client to problem-solve for herself and establish her own priorities. The nurse plans to administer a schedule dose of metoprolol (Toprol SR) at 0900 to a client with hypertension. At 0800, the nurse notes that client’s telemetry pattern shows a second degree heart block with a ventricular rate of 50. What action should the nurse take? • Administer the Tropol immediately and monitor the client until the heart rate increases. • Provide the dose of Tropol as scheduled and assign a UAP to monitor the client’s BP q30 minutes. • Give the Tropol as scheduled if the client’s systolic blood pressure reading is greater than 180. • Hold the scheduled dose of Tropol and notify the healthcare provider of the telemetry pattern. A client who developed syndrome of inappropriate antidiuretic hormone (SIADH) associated with small carcinoma of the lung is preparing for discharge. When teaching the client about self-management with demeclocycline (Declomycin), the nurse should instruct the client to report which condition to the health care provider? • Insomnia • Muscle cramping • Increase appetite • Anxiety. In determine the client position for insertion of an indwelling urinary catheter, it is most important for the nurse to recognize which client condition? • High urinary PH • Abdominal Ascites • Orthopnea • Fever. The nurse is reviewing a client’s electrocardiogram and determines the PR interval (PRI) is prolonged. What does this finding indicate? • Initiation of the impulses from a location outside the SA node • Inability of the SA node to initiate an impulse at the normal rate • Increased conduction time from the SA node to the AV junction • Interference with the conduction through one or both ventricles. The nurse is teaching a male client with multiple sclerosis how to empty his bladder using the Crede Method. When performing a return demonstration, the client applies pressure to the umbilical areas of his abdomen. What instruction should the nurse provide? • Stroke the inner thigh below the perineum to initiate urinary flow • Contract, hold, and then relax the pubococcygeal muscle • Pour warm water over the external sphincter at the distal glans • Apply downward manual pressure at the suprapubic regions. A 35 years old female client has just been admitted to the post anesthesia recovery unit following a partial thyroidectomy. Which statement reflects the nurse’s accurate understanding of the expected outcome for the client following this surgery? • Supplemental hormonal therapy will probably be unnecessary • The thyroid will regenerate to a normal size within a few years. • The client will be restricted from eating seafood • The remainder of the thyroid will be removed at a later date. A client with gestational diabetes, at 39 weeks of gestation, is in the second stage of labor. After delivering of the fetal head, the nurse recognizes that shoulder dystocia is occurring. What intervention should the nurse implement first? • Prepare the client for an emergency cesarean birth • Encourage the client to move to a hands-and-knees position. • Assist the client to sharply flex her thighs up again the abdomen. • Lower the head of the bed an apply suprapubic pressure. The nurse should observe most closely for drug toxicity when a client receives a medication that has which characteristic? • Low bioavailability • Rapid onset of action • Short half life • Narrow therapeutic index. Following insertion of a LeVeen shunt in a client with cirrhosis of the liver, which assessment finding indicates to the nurse that the shunt is effective? • Decrease abdominal girth • Increased blood pressure • Clear breath sounds • Decrease serum albumin. When finding a client sitting on the floor, the nurse calls for help from the unlicensed assistive personnel (UAP). Which task should the nurse ask the UAP to do? • Check for any abrasions or bruises. • Help the client to stand. • Get a blood pressure cuff. • Report the fall to the nurse-manager. During the initial newborn assessment, the nurse finds that a newborn's heart rate is irregular. Which intervention should the nurse implement? • Notify the pediatrician immediately. • Teach the parents about congenital heart defects. • Document the finding in the infant's record. • Apply oxygen per nasal cannula at 3 L/min. While performing a skin inspection for a female adult client, the nurse observes a rash that is well circumscribed, has silvery scales and plaques, and is located on the elbows and knees. These assessment findings are likely to indicate which condition? • Tinea corporis • Herpes zoster • Psoriasis • Drug reaction A client with acute pancreatitis is complaining of pain and nausea. Which interventions should the nurse implement (Select all that apply) • Monitor heart, lung, and kidney function. • Notify healthcare provider of serum amylase and lipase levels. • Review client’s abdominal ultrasound findings. • Position client on abdomen to provide organ stability • Encourage an increased intake of clear oral fluids A nurse is caring for a client with Diabetes Insipidus. Which assessment finding warrants immediate intervention by the nurse? • Hypernatremia • Excessive thirst • Elevated heart rate • Poor skin turgor The nurse weighs a 6-month-old infant during a well-baby check-up and determines that the baby’s weight has tripled compared to the birth weight of 7 pounds 8 ounces. The mother asks if the baby is gaining enough weight. What response should the nurse offer? • What food does your baby usually eat in a normal day? • What was the baby’s weight at the last well-baby clinic visit? • The baby is below the normal percentile for weight gain • Your baby is gaining weight right on schedule What is the nurse’s priority goal when providing care for a 2-year-old child experience… • Stop the seizure activity • Decrease the temperature • Manage the airway • Protect the body from injury The nurse is preparing to discharge an older adult female client who is at risk for hy…nurse include with this client’s discharge teaching? • Report any muscle twitching or seizures • Take vitamin D with calcium daily • Low fat yogurt is a good source of calcium • Keep a diet record to monitor calcium intake • Avoid seafood, particularly selfish The daughter of an older female client tells the clinic nurse that she is no longer able to care for her mother since her mother has lost the ability to perform activities of daily living (ADLs) due to aging. Which options should the nurse discuss with the daughter? • Home hospice agency • Long-term care facility • Rehabilitation facility • Independent senior apartment • Home health agency A male client with cancer, who is receiving antineoplastic drugs, is admitted to the…what findings is most often manifest this condition? • Ecchymosis and hematemesis • Weight loss and alopecia • Weakness and activity intolerance • Sore throat and fever A 7-year-old boy is brought to the clinic because of facial edema. He reports that he has been voiding small amounts of dark, cloudy, tea-colored urine. The parents state that their son had a sore throat 2 weeks earlier, but it has resolved. After assessing the child’s vital signs and weight, what intervention should the nurse implement next? • Measure the child’s abdominal girth • Perform an ostoscopic examination • Collect a urine specimen for routine urinalysis • Obtain a blood specimen for serum electrolytes The nurse observes an adolescent client prepare to administer a prescribed corticosteroid medication using a metered dose inhaler as seen in the picture. What action should the nurse take? • Remind the client to hold his breath after inhaling the medication • Confirm that the client has correctly shaken the inhaler • Affirm that the client has correctly positioned the inhaler • Ask the client if he has a spacer to use for this medication The nurse teaches an adolescent male client how to use a metered dose inhaler. Seen in the picture. What instruction should the nurse provide? • Move the device one to two inches away from the mouth • Secure the mouthpiece under the tongue • Press down on the device after breathing in fully • Breathe out slowly and deeply while compressing the device A 3-year-old boy with a congenital heart defect is brought to the clinic by his mother…During the assessment, the mother asks the nurse why her child is at the 5th percent…response is best for the nurse to provide? • Does your child seem mentally slower than his peers also? • “His smaller size is probably due to the heart disease” • Haven’t you been feeding him according to recommended daily allowances for children? • You should not worry about the growth tables. They are only averages for children A client with hypertension receives a prescription for enalapril, an angiotensin…instruction should the nurse include in the medication teaching plan? • Increase intake of potassium-rich foods • Report increased bruising of bleeding • Stop medication if a cough develops • Limit intake of leafy green vegetables When administering ceftriaxone sodium (Rocephin) intravenously to a client before…most immediate intervention by the nurse? • Stridor • Nausea • Headache • Pruritis The nurse is assessing a client with a small bowel obstruction who was hospitalized 24 hours ago. Which assessment finding should the nurse report immediately to the healthcare provider? • Rebound tenderness in the upper quadrants • Hypoactive bowel sounds in the lower quadrants • Tympany with percussion of the abdomen • Light colored gastric aspirate via the nasogastric tube An adult female client is admitted to the psychiatric unit because of a complex handwashing ritual she performs daily that takes two hours or longer to complete. She worries about staying clean and refuses to sit on any of the chairs in the day area. This client’s handwashing is an example of which clinical behavior? • Addiction • Phobia • Compulsion • Obsession Following routine diagnostic test, a client who is symptom-free is diagnosed with Paget’s disease. Client teaching should be directed toward what important goal for this client? • Maintain adequate cardiac output • Promote adequate tissue perfusion • Promote rest and sleep • Reduce the risk for injury After removing a left femoral arterial sheath, which assessment finding warrant immediately interventions by the nurse? (Select all that applied.) • Unrelieved back and flank pain. • Quarter-size red drainage at site • Cool and pale left leg and foot. • Tenderness over insertion site • Left groin egg-size hematoma. A client with urticaria due to an environmental allergies is taking diphenhydramine... Which complaint should the nurse identify to the client as a side effect of the OTC medication? • Nausea and indigestion. • Hypersalivation • Eyelid and facial twitching • Increased appetite In caring for a client with a PCA infusion of morphine sulfate through the right cephalic vein, The nurse assesses that the client in lethargic with a blood pressure of 90/60, pulse rate of 118 beats per minute, and respiratory rate of 8 breaths per minutes. What assessment should the nurse perform next? • Note the appearance and patency of the client’s peripheral IV site. • Palpate the volume of the client’s right radial pulse • Auscultate the client’s breath sounds bilaterally. • Observe the amount and dose of morphine in the PCA pump syringe. Which instruction is most important for the nurse to provide a client who is being discharge following treatment for Guillain-Barre syndrome? • Avoid exposure to respiratory infections • Use relaxation exercises when anxious • Plan short, frequent rest periods • Continue physical therapy at home The nurse assesses a female client with obstructive sleep apnea syndrome (OSAS) who is 5 feet tall (152 cm) and weighs 155 pounds (70 kg), the client’s 24 hour diet history includes: no breakfast, cheeseburger and fries for lunch; lasagna, chocolate ice cream and a cola drink for dinner, and 2 glasses of wine in the evening before going to bed for a total caloric intake of 3500 calories. What instructions should the nurse provide? (Select all that apply) • Maintain current caloric intake • Avoid use of alcohol as a sleep aide at bedtime • Reduce intake of dairy products • Start a weight loss program • Set a goal of increasing BMI (Body Mass Index) The health care provider prescribes atenolol 50 mg daily for a client with angina pectoris…to the health care provider before administering this medication? • Irregular pulse • Tachycardia • Chest pain • Urinary frequency A male Korean-American client looks away when asked by the nurse to describe his problem. What is the best initial nursing action? • Allow several minutes for the client to respond • Ask social services to find a Korean interpreter • Repeat the question slowly and distinctly • Establish direct eye contact with the client The nurse provides feeding tube instructions to the wife of a client with end stage cancer. The client’s wife performs a return demonstration correctly, but begins crying and tells the nurse, “I just don’t think I can do this every day.” The nurse should direct further teaching strategies toward which learning domain? • Cognitive • Affective • Comprehension • Psychomotor A woman who takes pyridostigmine for myasthenia gravis (MG) arrives at the emergency department complaining of extreme muscle weakness. Her adult daughter tells the nurse that since yesterday her mother has been unable to smile, which assessment finding warrants immediate intervention by the nurse? • Uncontrollable drooling • Inability to raise voice • Tingling of extremities • Eyelid drooling What is the nurse’s priority goal when providing care for a 2-year-old child experiencing seizure… • Stop the seizure activity • Decrease the temperature • Manage the airway • Protect the body from injury The nurse is caring for four clients…postoperative hemoglobin of 8.7 mg/dl; client C, newly admitted with potassium…an appendectomy who has a white blood cell count of 15,000mm3. What intervention… • Determine the availability of two units of packed cells in the blood bank for client B • Increase the oxygen flow rate to 4 liters/minute per face mask for client A • Remove any foods, such as banana or orange juice, for the breakfast tray for client C • Inform client D that surgery is likely to be delayed until the infection responds to antibiotics While the school nurse is teaching a group of 14-year-olds, one of the participants remarks, “You are too young to be our teacher! You’re not much older than we are!” How should the nurse respond? • “How old do you think I am?” • “We need to stay focused on the topic.” • “I think I am qualified to teach this group.” • “Do you think you can teach it any better?” A middle-aged woman, diagnosed with Graves’ disease, asks the nurse about this condition. Which etiological pathology should the nurse include in the teaching plan about hyperthyroidism? (Select all that apply.) • Graves’ disease, an autoimmune condition, affects thyroid stimulating hormone receptors. • T3 and T4 hormone levels are increased • Large protruding eyeballs are a sign of hyperthyroid function • Weight gain is a common complaint in hyperthyroidism • Early treatment includes levothyroxine (Synthroid). When assessing a multigravida the first postpartum day, the nurse finds a moderate amount of lochia rubra, with the uterus firm, and three fingerbreadths above the umbilicus. What action should the nurse implement first? • Massage the uterus to decrease atony • Review the hemoglobin to determine hemorrhage • Increase intravenous infusion • Check for a distended bladder A 59-year-old male client comes to the clinic and reports his concern over a lump that, “just popped up on my neck about a week ago.” In performing an examination of the lump, the nurse palpates a large, nontender, hardened left subclavian lymph node. There is not overlying tissue inflammation. What do these findings suggest? • Malignancy • Bacterial infection • Viral infection • Lymphangitis A gravida 2 para 1, at 38-weeks gestation, scheduled for a repeat cesarean section in one week, is brought to the labor and delivery unit complaining of contractions every 10 minutes. While assessing the client, the client’s mothers enter the labor suite and says in a loud voice, “I’ve had 8 children and I know she’s in labor. I want her to have her cesarean section right now!” what action should the nurse take? • Request the mother to leave the room • Tell the mother to stop speaking for the client • Request security to remove her from the room • Notify the charge nurse of the situation While caring for a toddler receiving oxygen (02) via face mask, the nurse observes that the child’s lips and nares are dry and cracked. Which intervention should the nurse implement? • Ask the mother what she usually uses on the child’s lips and nose • Apply a petroleum jelly (Vaseline) to the child’s nose and lips • Use a topical lidocaine (Zylocaine viscous) analgesic for cracked lips • Use a water soluble lubricant on affected oral and nasal mucosa The healthcare provider prescribes carboprost tromethamine (Hemabate) 250 mcg IM for a multigravida postpartum client who is experiencing heavy, bright red vaginal bleeding. Prior to administering this medication, which interventions should the RN implement? • Obtain a second IV access. • Decrease the room temperature. • Give the prescribed antiemetic. • Insert an indwelling catheter. During the infusion of a second unit of packed red blood cells, the client’s temperature increases from 99 to 101.6 f. which intervention should the nurse implement? • Stop the transfusion start a saline • Observe for a maculopapular rash • Report the fever to the blood bank • Give a PRN dose of acetaminophen During discharge teaching, an overweight client heart failure (HF) is asked to make a grocery list for the nurse to review. Which food choices included on the client’s list should the nurse encourage? (Select all that apply) A. Canned fruit in heavy syrup. B. Plain, air-popped popcorn. C. Cheddar cheese cubes. D. Natural whole almonds. E. Lightly salted potato chips A client with Addison’s disease becomes weak, confused, and dehydrated following the onset of an acute viral infection. The client’s laboratory values include; sodium 129 mEq/l (129mmol/l SI), glucose 54 mg/dl (2.97mmol/l SI) and potassium 5.3 mmol/l SI). When reporting the findings to the HCP, the nurse anticipates a prescription for which intravenous medications? A. Regular insulin. B. Hydrocortisone C. Broad spectrum antibiotic D. Potassium chloride An adolescent, whose mother recently died, comes to the school nurse complain headache. Which statement made by the students should warrant further explanation nurse? A. “I’ve had dreams about Mon since she died.” B. “I’ve been very sad and cry a lot at night.” C. “I miss Mon and would like to go see her’”. D. “ it’s hard to concentrate on my homework” The nurse is caring for four clients who are on the rehabilitation unit, which client should the nurse assess first? A. A client with an above-the-knee amputation who is complaining of phantorn pain. B. A client who is receiving a continuous tube feeding and is now vomiting. C. A client with left hemiplegia who is scheduled for hemodialysis today. D. A client with pneumonia who is scheduled for pulmonary function studies. A client’s telemetry monitor indicates ventricular fibrillation (VF). After delivering one counter shock, the nurse resumes chest compression, after another minute of compression , the client’s rhythm converts to supraventricular tachycardia (SVT) on the monitor, at this point , what is the priority intervention for the nurse? A. Prepare for transcutaneous pacing B. Administer IV epinephrine per ACLS protocol. C. Give IV dose of adenosine rapidly over 1-2 seconds. D. Deliver another defibrillator shock. A client with a history of using illicit drugs intravenously is admitted with Kaposi’s sarcoma. Which intervention should the nurse include in this client’s admission plan of care? A. Identify local support HIV support groups. B. Assess for symptoms of AIDS dementia. C. Observe for adverse drug reaction. D. Monitor for secondary infections. After an elderly female client receives treatment for drug toxicity, the HCP prescribes a 24- hour creatinine clearance test. Prior to starting the urine collection, the nurse notes that the client’s serum creatinine is 0.3mg/dl. What action should the nurse implement? A. Initiate the urine collection as prescribed. B. Notify the HCP of the results. C. Evaluate the client’s serum BUN level. D. Assess the client for signs of hypokalemia. Immediately after extubation, a client who has been mechanically ventilated is placed on a 50% non-rebreather. The client is hoarse and complaining of a sore throat. Which assessment finding should the nurse report to the healthcare provider immediately? A. Blood tinged sputum B. Expiratory wheezing C. Upper airway stridor D. Oxygen saturations 90% The nurse is collecting sterile sample for culture and sensitivity from a disposable three chamber water-seal drainage system connected to a pleural chest tube. The nurse should obtain the sample from which site on the drainage system? A. Stopper port located above the water-seal level B. Plastic tubing located at the chest insertion site C. Rubberized port at the bottom of collection chamber D. Tubbing located on the top of the suction chamber While the nurse is preparing a scheduled intravenous (IV) medication, the client states that the IV site hurts and refuses to allow the nurse to administer a flush to assess the site. Which intervention should the nurse implement? A. Apply ice first, then a warm compress to the IV site B. Discontinue the painful IV after a new IV is inserted C. Review the medical record for the date of insertion D. Document that the medication was not administered During a staff meeting, a nurse verbally attacks the nurse manager conducting the meeting, stating, “you always let your favorites have holidays off give then easier assignments. You are unfair and prejudiced” how should the nurse-manager respond? A. I would prefer to discuss this with you privately. B. Give me specific examples to support your statements. C. Does anyone else on the staff fell the same way D. Your remarks are not true and are very unkind An adult is admitted to the emergency department following ingestion of a bottle of antidepressants secondary to chronic paint. A nasogastric tube and a left subclavian venous catheter are placed. The nurse auscultates audible breath sounds on the right side, faint sounds procedure should the nurse prepare for first? A. Insertion of a left- sided chest tube. B. Placement of an endotracheal tube. C. Retraction of the nasogastric tube D. Setup of patient- controlled analgesia A client is admitted to the hospital after experiencing a brain attack, commonly referred to as a stroke or cerebral vascular accident (CVA). The nurse should request a referral for speech therapy if the client exhibits which finding? A. Abnormal responses for cranial nerves I and II B. Persistent coughing while drinking C. Unilateral facial drooping D. Inappropriate or exaggerated mood swings A male client is admitted with a severe asthma attack. For the last 3 hours he has experienced increased shortness of breath. His arterial blood gas results are: pH 7.22 PaCO2 55 mmHg; HCO3 25 mEq/L or mmol/L (SI). Which intervention should the nurse implement? A. Space care to provide periods of rest B. Instruct client to purse lip breathe C. Administer PRN dose of albuterol D. Position client for maximum comfort A young adult female with chronic kidney disease (CKD) due to recurring pyelonephritis is hospitalized with basilar crackles and peripheral edema. She is complaining of severe nausea and the cardiac monitor indicates sinus tachycardia with frequent premature ventricular contraction. Her blood pressure is 200 /110 mm Hg, and her temperature is 101 F which PRN medication should the nurse administers first? A. Enalapril B. Furosemide C. Acetaminophen D. Promethazine When entering a client’s room to administer an 0900 IV antibiotic, the nurse finds that the client is engaged in sexual activity with a visitor. Which actions should the nurse implement? A. Ignore the behavior and hang the IV antibiotic B. tell the client to stop the inappropriate behavior C. Leave the room and close the door quietly D. Complete an unusual occurrence report A child with heart failure (HF) is taking digitalis. Which signs indicates to the nurse that the child may be experiencing digitalis toxicity? A. Tachycarcia B. Dyspnea C. Vomiting D. Muscle cramps An antacid is prescribed for a client with gastroesophageal (GERD). The client asks the nurse, “How does this help my GERD?” What is the best response by the nurse? A. This medication will coat the lining of your esophagus B. Antacids will neutralize the acid in your stomach C. It will improve the emptying of food through your stomach D. antacids decrease the production of gastric secretions A male adult is admitted because of an acetaminophen overdose. After transfer to the mental health unit, the client is told he has liver damage. Which information is most important for the nurse to include in the client’s discharge plan? A. Avoid exposure to large crowds B. Do not take any over-the-counter medications C. Call the crisis hot line if feeling lonely D. Eat a high carbohydrate, low fat, low protein diet A client arrives in the emergency center with a blood alcohol level of 500 mg/dl. When transferred to the observation unit, the client becomes demanding, aggressive, and shouts at the staff. Which assessments finding is most important for the nurse to identify in the first 24 hours? A. Decreased appetite B. Nausea and elevated blood pressure C. Difficulty walking D. Agitation and threats to harms staff To reduce the risk of symptoms exacerbation for a client with multiple sclerosis (MS), which instructions should the nurse include in the client’s discharge plan? (Select all that apply). A. Practice relaxation exercises B. Limit fluids to avoid bladder distention C. Space activities to allow for rest periods D. Avoid persons with infections E. Take warm baths before starting exercise A preoperative client states he is not allergic to any medications. What is the most important nursing action for the nurse to implement next? A. Record “no known drug allergies” on preoperative checklist B. Assess client’s allergies to non-drug substances C. Assess client’s knowledge of an allergy response D. Flag “no known drug allergies” on the front of the chart During a visit to the planned parenthood clinic, a young woman tells the nurse that she is going to discontinue taking the oral contraceptives she has taken for three years because she wants to get pregnant. History indicates that her grandfather has adult onset diabetes and that she was treated for chlamydia six months ago, which factor in this client’s history poses the greatest risk for this woman’s pregnancy? A. Family history of adult onset diabetes. B. Treatment for chlamydia in the past year C. Client’s age and previous sexual behavior D. Three year history of taking oral contraceptives When conducting diet teaching for a client who was diagnosed with a myocardial infarction, which snack foods should the nurse encourage the client to eat? (Select all that apply). A. Fresh turkey slices and berries B. Fresh vegetables with mayonnaise dip C. Soda crackers and peanut butter D. Chicken bouillon soup and toast E. raw unsalted almonds and apples A mother brings her 3-week-old son to the clinic because he is vomiting “all the time.” In performing a physical assessment, the nurse notes that the infant has poor skin turgor, has lost 20% of his birth weight, and has a small palpable oval-shaped mass in his abdomen. What intervention should the nurse implement first? A. Give the infant 5% dextrose in water orally B. Insert a nasogastric tube for feeding C. Initiate a prescribed IV for parental fluid D. Feed the infant 3 ounces of Isomil An older woman who was recently diagnosed with end stage metastatic breast cancer is admitted because she is experiencing shortness of breath and confusion. The client refuses to eat and continuously asks to go home. Arterial blood gases indicate hypoxia. Which intervention is most important for the nurse to implement? A. Prepare for emergent oral intubation B. Offer sips of favorite beverages C. Clarify end of life desires D. Initiate comfort measures The nurse is triaging several children as they present to the emergency room after an accident. Which child requires the most immediate intervention by the nurse? A. A 12-year-old with complaints of neck and lower back discomfort B. An 11-year-old with a headache, nausea, and projectile vomiting C. A 6-year-old with multiple superficial lacerations of all ectremities D. An 8-year-old with a full leg air splint for a possible broken tibia An unlicensed assistive personnel (UAP) reports that a client’s right hand and fingers spasms when taking the blood pressure using the same arm. After confirming the presence of spams what action should the nurse take? A. Ask the UAP to take the blood pressure in the other arm B. Tell the UAP to use a different sphygmomanometer. C. Review the client’s serum calcium level D. Administer PRN antianxiety medication. The nurse is caring for a 17-year-old male who fell 20 feet 5 months ago while climbing the side of a cliff and has been in a sustained vegetative state since the accident. Which intervention should the nurse implement? A. Inquire about food allergies and food likes and dislikes B. Talk directly to the adolescent while providing care C. Initiate open communication with the teen’s parents D. Monitor vital signs and neuro status every 2 hours Following an open reduction of the tibia, the nurse notes bleeding on the client’s cast. Which action should the nurse implement? A. No action is required since postoperative bleeding can be expected B. Lower the client’s head while assessing for symptoms of shock C. Call the health care provider and prepare to take the client back to the operating room D. Outline the area with ink and check it every 15 minutes to see if the area has increased While a child is hospitalized with acute glomerulonephritis, the parents ask why blood pressure readings are taken so often. Which response by the nurse is most accurate? A. Blood pressure fluctuations means that the condition has become chronic B. Elevated blood pressure must be anticipated and identified quickly C. Hypotension leading to sudden shock can develop at any time D. Sodium intake with meals and snacks affects the blood pressure An elderly male client is admitted to the urology unit with acute renal failure due to a post-renal obstruction. Which questions best assists the nurse in obtaining relevant historical data? A. “Have you had a heart attack in the last 6 months” B. “Have you had any difficulty in starting your urinary stream” C. “Have you taken any antibiotics recently” D. “Have you received any blood products in the last year” A child is diagnosed with acquired aplastic anemia. The nurse knows that this child has the best prognosis with which treatment regimen? A. Bone marrow transplantation B. Blood transfusion C. Chemotherapy D. Immunosuppressive therapy A client is admitted with an exacerbation of heart failure secondary to COPD. Which observations by the nurse require immediate intervention to reduce the likelihood of harm to this client? (Select all that apply). A. A bedside commode is positioned near the bed B. A saline lock is present in the right forearm C. A full pitcher of water is on the bedside table D. The client is lying in a supine position in bed E. A low sodium diet tray was brought to the room A client with a traumatic brain injury becomes progressively less responsive to stimuli. The client has a “Do Not Resuscitate” prescription, and the nurse observes that the unlicensed assistive personnel (UAP) has stopped turning the client from side to side as previously schedules. What action should the nurse take? A. Advise the UAP to resume positioning the client on schedule B. Encourage the UAP to provide comfort care measures only C. Assume total care of the client to monitor neurologic function D. Assign a practical nurse to assist the UAP in turning the client The nurse reviews the laboratory findings of a client with an open fracture of the tibia. The white blood cell (WBC) count and erythrocyte sedimentation rate (ESR) are elevated. Before reporting this information to the healthcare provider, what assessment should the nurse obtain? A. Degree of skin elasticity B. Appearance of wound C. Bilateral pedal pulse force D. Onset of any bleeding A client is admitted to the surgical unit with symptoms of a possible intestinal obstruction. When preparing to insert a nasogastric (NG) tube, which intervention should the nurse implement? A. Elevate the head of the bed 60 to 90 degrees B. Measure from corner of mouth to angle of jaw C. Administer a PRN analgesic D. Assess for a gag reflex In early septic shock states, what is the primary cause of hypotension? A. Cardiac failure B. A vagal response C. Peripheral vasoconstriction D. Peripheral vasodilation The charge nurse observes a new nurse preparing to insert an intravenous (IV) catheter. The new nurse has gathered supplies, including intravenous catheters, an intravenous insertion kit, and a 4x4 sterile gauze dressing to cover and secure the insertion site. What action should the charge nurse take? A. Plan to observe the secured IV site after the insertion procedure B. Confirm that the nurse has gathered the necessary supplies C. Remind the nurse to tape the gauze dressing securely in place D. Instruct the nurse to use a transparent dressing over the site An adult client comes to the clinic and reports his concern over a lump that “just popped up on my neck about a week ago.” In performing an examination of the lump, the nurse palpates a large, non-tender, hardened left subclavian lymph node. There is no overlying tissue inflammation. What do these finding suggest? A. Bacterial infection B. Lymphangitis C. Malignancy D. Viral infection The nurse is preparing to administer an IV dose of ciprofloxacin to a client with urinary tract infection. Which client data requires the most immediate intervention by the nurse? A. Urine culture positive for MRSA B. Serum sodium of 145 mEq/L (145 mmol/L SI) C. Serum creatinine of 4.5 mg/dl (398 mcmol/L SI) D. White blood cell count of of 12,000 mm3 (12 x 109/L SI) The unit clerk reports to the charge nurse that a healthcare provider has written several prescriptions that are illegible and it appears the healthcare provider used several unapproved abbreviations in the prescriptions. What actions should the charge nurse take? A. Complete and file an incident (variance) report B. Call the healthcare provider who wrote the prescription C. Contact the healthcare provider review board for instructions D. Report the situation to the house supervisor A confused, older client with Alzheimer’s disease becomes incontinent of urine when attempting to find the bathroom. Which action should the nurse implement? A. Instruct the client to use the call button when a bedpan is needed B. Apply adult diapers after each attempt to void C. Check residual urine volume using an indwelling urinary catheter D. Assist the client’s to a bedside commode every two hours The nurse is caring for a group of clients with the help of a practical nurse (PN). Which nursing actions should the nurse assign to the PN? (Select all that apply.) A. Administer a dose of insulin per sliding scale for a client with type 2 diabetes mellitus (DM). B. Obtain postoperative vital signs for a client one day following unilateral knee arthroplasty C. Perform daily surgical dressing change for a client who had an abdominal hysterectomy D. Initiate patient controlled analgesia (PCA) pumps for two clients immediately postoperative E. Start the second blood transfusion for a client twelve hours following a below knee amputation In caring for a client who is receiving linezolid IV for nosocomial pneumonia, which assessment finding is most important for the nurse to report to the healthcare provider? A. Watery diarrhea B. Yellow-tinged sputum C. Increased fatigue D. Nausea and headache An 11-year-old client is admitted to the mental health unit after trying to run away from home and threatening self-harm. The nurse establishes a goal to promote effective coping, and plans to ask the client to verbalize three ways to deal with stress. Which activity is best to establish rapport and accomplish this therapeutic goal? A. Bring the client to the team meeting to discuss the treatment plan B. Ask the client to write feeling in a journal and then review it together C. Explain the purpose of each medication the client is currently taking D. Play a board game with the client and begin taking about stressors An adult male with schizophrenia who has been noncompliant in taking oral antipsychotic medications refuses a prescribed IM medication. Which action should the nurse take? A. Notify the healthcare provider of the client’s refusal B. Administer an oral PRN medication for agitation C. Ask for staff assistance with administering the injection D. explain that oral medications will no longer be required An older male client with a history of diabetes mellitus, chronic gout, and osteoarthritis comes to the clinic with a bag of medication bottles. Which intervention should the nurse implement first? A. Record pain evaluation B. Assess blood glucose C. Identify pills in the bag D. Obtain a medical history A male client with an antisocial personality disorder is admitted to an in-patient mental health unit for multiple substance dependency. When providing a history, the client justifies to the nurse his use of illicit drugs. Based on this pattern of behavior this client’s history is most likely to include which finding? A. Phobias and panic attacks when confronted by authority figures. B. Suicidal ideations and multiple attempts/ C. Multiple convictions for misdemeanors and class B felonies. D. Delusions of grandiosity and persecution An adult male who fell from a roof and fractures his left femur is admitted for surgical stabilization after having a soft cast applied in the emergency department. Which assessment finding warrants immediate intervention by the nurse? A. Onset of mild confusion B. Pain score 8 out of 10 C. Pale, diaphoretic skin D. Weak palpable distal pulses A client who has a suspected brain tumor is schedules for a computed (CT) scan. When preparing the client for the client for the CT scan, which intervention should the nurse implement? A. Determine if the client has had a knee or hip replacement B. Immobilize the client’s neck before moving onto stretcher C. Give an antiemetic to control nausea D. Obtain the client’s food allergy history A client who is at 10-weeks gestation calls the clinic because she has been vomiting for the past 24 hours. The nurse determines that the client has no fever. Which instructions should the nurse give to this client? A. Remain on clear liquids until the vomiting subsides B. Come to the clinic to be seen by a healthcare provider C. Make an appointment at the clinic if a fever occurs D. Take nothing by mouth until there is no more nausea The nurse is preparing to gavage feed a premature infant through an orogastric tube. During insertion of the tube, the infant’s heart rate drops to 60 beats / minute. Which action should the nurse take? A. Continue the insertion since this is a typical response B. pause and monitor for a continues drop of the heart rate C. Insert the feeding tube into the infant’s nasal passage D. Postpone the feeding until the infant’s vital signs and stable A male client is admitted with a bowel obstruction and intractable vomiting for the last several hours despite the use of antiemetics. Which intervention should the nurse implement first? (Please scroll and view each tab’s information in the client’s medical record before selecting the answer.) pH 7.5 PaCO2 42 mmHG, HCO3 33 mEq/l, pO2 92 mmHg A. Infuse 0.9 % sodium chloride 500 ml bolus B. Insert nasogastric tube to intermittent suction. C. Maintain head of bed at 45 degrees D. Document strict intake and output A client is being treated for syndrome of inappropriate antidiuretic hormone (SIADH). On examination, the client has a weight gain of 4.4 lbs (2 kg) in 24 hours and an elev ated blood pressure. Which intervention should the nurse implement first? A. Ensure client takes a diuretic q AM B. Obtain serum creatinine levels daily C. Measure ankle circumference D. Monitor daily sodium intake The nurse and an unlicensed assistive personnel (UAP) are providing care for a client with a nasogastric tube (NGT) when the client begins to vomit. How should the nurse manage this situation? A. Determine the presence of hematemesis as the UAP irrigates the NGT B. Instruct the UAP to bring an antiemetic to the nurse at the bedside C. Assess the appearance of the emesis while the UAP checks bowel sounds D. Direct the UAP to measure the emesis while the nurse irrigates the NGT A preschooler with constipation needs to increase fiber intake. Which snack suggestion should the nurse provide? A. soft pretzels B. fruit-flavored yogurt C. oatmeal cookies D. low fat cheese sticks After multiple attempts to stop drinking, an adult male is admitted to the medical intensive care unit (MICU) with delirium tremens. He is tachycardic, diaphoretic, restless, and disoriented. Which finding indicates a life- threatening condition? A.CIWA-Ar for alcohol withdrawal score of 30 B.Acute onset of unrelenting chest pain C. Widening QRS complexes and flat waves D. Intense tremor and involuntary muscle activity The home health nurse is preparing to make daily visits to a group of clients. Which client should the nurse visit first? A. A client with congestive heart failure who reports a 3 pound weight gain in the last two days B. An immobile client with a stage 3 pressure ulcer on the coccyx who is having low back pain C. A client diagnosed with chronic obstructive pulmonary disease (COPD) who is short of breath D. A terminally ill older adult who has refused to eat or drink anything for the last 48 hours A female client is admitted for diabetic crisis resulting from inadequate dietary practices. After stabilization, the nurse talks to the client about her prescribed diet. What client characteristic is most import for successful adherence to the diabetic diet? A. Knows that insulin must be given 30 min before eating B. Frequently eats fruits and vegetables at meals and between meals/ C. Has someone available who can prepare and oversee the diet D. Demonstrates willingness to adhere to the diet consistently Oxygen at 5l/min per nasal cannula is being administered to a 10 year old child with pneumonia. When planning care for this child, what principle of oxygen administration should the nurse consider? A. Taking a sedative at bedtime slows respiratory rate, which decreases oxygen? B. Avoid administration of oxygen at high levels for extendedperiods. C. Increase oxygen rate during sleep to compensate for slower respiratory rate. D. Oxygen is less toxic when it is humidified with a hydration source. The nurse is caring for a client with acute kidney injury (AKI) secondary to gentamicin therapy the client’s serum blood potassium is elevated, which finding requires immediate action by the nurse? A. Tall peak T waves on the cardiac monitor B. Peripheral pitting edema at 2 + indentation C. Serum creatinine above 0.5 mg/dl or 44.2 micro-mmol/dl D. Anuria for the last 12 hours. During orientation, a newly hired nurse demonstrates suctioning of a tracheostomy in a skills class, as seen in the video. After the demonstration, the supervising nurse expresses concern that the demonstrated procedure increased the client’s risk for which problem? A. Infection B. Ineffective airway clearance C. Altered comfort D. Impaired gas exchange A male client with diabetes mellitus type 2, who is taking pioglitazone PO daily, reports to the nurse the recent onset of nausea, accompanied by dark-colored urine, and a yellowish cast to his skin. What instructions should the nurse provide? A. “You have become dehydrated from the nausea. You will need to rest and increase fluid intake” B. “you need to seek immediate medical assistance to evaluate the cause of these symptoms” C. A urine specimen will be needed to determine what kind of infection you have developed” D. use insulin per sliding scale until the nausea resolves, and then resume your oral medication” A male client with ulcerative colitis received a prescription for a corticosteroid last month, but because of the side effect he stopped taking the medication 6 year ago. Which finding warrants immediate intervention by the nurse? A. Hypotension and fever B. Anxiety and restlessness. C. Fluid retention D. Increased blood glucose. A client in the intensive care unit is being mechanically ventilated, has an indwelling urinary catheter in place, an exhibiting signs of restlessness. Which action should the nurse take fist? A. Administer PRN dose of lorazepam B. Auscultate bilateral breath sounds C. Check urinary catheter for obstruction D. Review the heart rhythms on cardiac monitor. A young adult female client with recurrent pelvic pain for 3 year returns to the clinic for relief of severe dysmenorrhea. The nurse reviews her medical record which indicates that the client has endometriosis. Based on this finding, what information should the nurse provide this client? A) Oral contraceptives increase the symptoms of endometriosis. B) The symptoms of endometriosis can increase with menopause. C) An option to diagnose disease extent and provide therapeutic treatment is laparoscopy. D) Infertile is successfully treated with removal of intra-abdominal endometrial lesions. A 75-year-old female client is admitted to the orthopedic unit following an open reduction and internal fixation of a hip fracture. On the second postoperative day, the client becomes confused and repeatedly asks the nurse she is. What information for the nurse to obtain? A. Use of sleeping medications. B. History of alcohol use, C. Use of antianxiety medications, D. History of this behavior. To reduce the risk of being named in malpractice lawsuit, which action is most important for the nurse to take? A. Establish a trusting nurse-client relationship. B. Complete an incident report following a client injury. C. Maintain current professional malpractice insurance, D. Adhere consistently to standards of care. A client with multiple sclerosis is receiving beta-1b interferon every other day. To assess for possible bone marrow suppression caused by the medication, which serum laboratory test findings should the nurse monitor? (Select all that apply) A. Platelet count B. Red blood cell count (RBC) C. White blood cell count (WBC). D. Albumin and protein E. Sodium and potassium The nurse is auscultating a client’s lung sounds. Which description should the nurse use to document this sound? (Please listen to the audio file to select the option that applies.) https://www.youtube.com/watch?v=VGDdqtIhUdA A. High pitched or fine crackles. B. High pitched wheeze C. Rhonchi D. Stridor The nurse is assessing and elderly bedridden client. Which finding indicates that the turning and positioning schedule is effective in protecting the client’s skin? A. Reddened skin areas disappear within 15 minutes of being turned and positioned. B. No complaints of pressure or pain are verbalized by the client after being turned C. Only small areas of redness remain longer than 30 min after the client is turned. D. The client verbalizes feeling better after being turned and positioned A client with a liver abscess develops septic shock. A sepsis resuscitation bundle protocol is initiated and the client receives a bolus of IV fluids. Which parameter should the nurse monitor to assess effectiveness of the fluid bolus? A. Mean arterial pressure (MAP) B. White blood cell count C. Blood culture D. Oxygen saturation A 17-year –old male is brought to the emergency department by his parents because he has been coughing and running a fever with flu-like symptoms for the past 24 hours. Which intervention should the nurse implement first? A. Obtain a chest X-ray per protocol. B. Place a mask on the client’s face. C. Assess the client’s temperature. D. Determine the client’s blood pressure An older client is admitted for repair of a broken hip. To reduce the risk for infection in the postoperative period, which nursing care interventions should the nurse include in the client’s plan of care? (Select all that apply) A. Teach client to use incentive spirometer q2 hours while awake. B. Remove urinary catheter as soon as possible and encourage voiding. C. Maintain sequential compression devices while in bed. D. Administer low molecular weight heparin as prescribed E. Assess pain level and medicate PRN as prescribed. A client is scheduled to receive an IW dose of ondansetron (Zofran) eight hours after receiving chemotherapy. The client has saline lock and is sleeping quietly without any restlessness. The nurse caring for the client is not certified in chemotherapy administration. What action should the nurse take? A. Ask a chemotherapy-certified nurse to administer the Zofran B. Administer the ondasentron (Zofran) after flushing the saline lock with saline C. Hold the scheduled dose of Zofran until the client awakens D. Awaken the client to assess the need for administration of the Zofran. While visiting a female client who has heart failure (HF) and osteoarthritis, the home health nurse determines that the client is having more difficulty getting in and out of the bed than she did previously. Which action should the nurse implement first? A. Inquire about an electric bed for the client’s home use B. Submit a referral for an evaluation by a physical therapist. C. Explain the usual progression of osteoarthritis and HF D. Request social services to review the client’s resources. A client is admitted to a mental health unit after attempting suicide by taking a handful of medications. In developing a plan of care for this client, which goal has the highest priority? A. Signs a no-self-harm contract. B. Sleep at least 6 hours nightly. C. Attends group therapy every day D. Verbalizes a positive self-image. The nurse is ready to insert an indwelling urinary catheter as seen in the picture. At this point in the procedure, what actions should the nurse take before inserting the catheter? (Select all that apply) picture 14 A. Ask the client to bear down as if voiding to relax the sphincter B. Complete perianal care with soap and water C. Gently palpate the client’s bladder for distention D. Hold the catheter 3 – 4 inches (7.5 – 10 cm) from its tip E. Secure the urinary drainage bag to the bed frame [Show More]

Last updated: 1 year ago

Preview 1 out of 91 pages

Reviews( 0 )

$20.00

Add to cart

Instant download

Can't find what you want? Try our AI powered Search

OR

GET ASSIGNMENT HELP
86
0

Document information


Connected school, study & course


About the document


Uploaded On

Dec 26, 2020

Number of pages

91

Written in

Seller


seller-icon
Good grade

Member since 4 years

395 Documents Sold


Additional information

This document has been written for:

Uploaded

Dec 26, 2020

Downloads

 0

Views

 86

Document Keyword Tags

Recommended For You


$20.00
What is Browsegrades

In Browsegrades, a student can earn by offering help to other student. Students can help other students with materials by upploading their notes and earn money.

We are here to help

We're available through e-mail, Twitter, Facebook, and live chat.
 FAQ
 Questions? Leave a message!

Follow us on
 Twitter

Copyright © Browsegrades · High quality services·